ADN 120 Final exam Silvestri Questions

Pataasin ang iyong marka sa homework at exams ngayon gamit ang Quizwiz!

A 15-year-old female seeks treatment for a sexually transmitted infection at a local clinic. With regard to informed consent, the nurse should perform which action? A. Ask the client to sign the informed consent form. B. Tell the client that a court order for treatment is needed. C. Tell the client that parental consent for treatment is needed. D. Call the client's mother to obtain telephone consent for treatment.

A. Ask the client to sign the informed consent form.

A client is being treated for anorgasmia. Which assessment question demonstrates that the nurse understands the likely causes of the disorder? A. "How would you feel if you became pregnant?" B. "When did you have your first menstrual period?" C. "Do you feel you eat a well-balanced, nutritious diet?" D. "Does depression seem to affect members of your family?"

A. "How would you feel if you became pregnant?"

What are the main groups of schizophrenia symptoms? Select all that apply. A. Positive B. Affective C. Negative D. Cognitive E. Distributive F. Accumulative

A. Positive B. Affective C. Negative D. Cognitive

The nurse notes documentation in a client's medical record that the client is experiencing anuria. Based on this notation, what determination should the nurse make? A. The client is unable to produce urine. B. The client has a diminished capacity to form urine. C. The client has difficulty having a bowel movement. D. The client has episodes of alternating constipation and diarrhea.

A. The client is unable to produce urine.

A client presents to the emergency department with the following assessment: awake, alert, confused, and obeys commands, and moves all four extremities. Eyes open spontaneously. The client exhibits a verbal response to questions asked. What is the calculated Glasgow Coma Scale (GCS) score for these assessment findings? A. 15 B. 14 C. 12 D. 10

B. 14

The nurse receives the assigned clients for the day. With which client should the nurse follow up first? A. A client with emphysema has a pulse oximetry level of 91%. B. A client on chemotherapy has a temperature of 100.4° F (38° C). C. A client receiving external beam radiation therapy has reddened skin at the site. D. A client with chronic obstructive pulmonary disease (COPD) has prolonged expiration (1:4 ratio).

B. A client on chemotherapy has a temperature of 100.4° F (38° C).

The nurse is reviewing a client's admission laboratory results and notes that the client's serum calcium level is 14 mg/dL (3.5 mmol/L). The nurse should check to see that which medication that may be prescribed is available in the stock medication supply area on the clinical nursing unit? A. Vitamin D B. Calcitonin C. Calcium chloride D. Calcium gluconate

B. Calcitonin

The nurse is performing an assessment on a client admitted to the hospital with a diagnosis of dehydration. Which assessment finding would the nurse expect to note? A. Bradycardia B. Changes in mental status C. An elevated blood pressure D. Bilateral crackles in the lungs

B. Changes in mental status

A client receiving a blood transfusion develops signs of a blood transfusion reaction. The nurse stops the transfusion and maintains the intravenous (IV) line with normal saline. Which action should the nurse take next? A. Document the occurrence. B. Check the client's vital signs. C. Send the blood bag and tubing to the blood bank for examination. D. Check the client's urine output, and obtain a urine specimen for analysis.

B. Check the client's vital signs.

The client is undergoing induction of labor at 41 weeks' pregnancy. The nurse midwife has placed dinoprostone in the posterior fornix of the vagina for cervical ripening. Which finding would require the nurse to notify the health care provider? A. Client complaint of cramping B. Contractions occurring every minute C. Vaginal discharge of mucus mixed with blood D. Fetal heart rate of 120 beats/min

B. Contractions occurring every minute

The client has sickle cell anemia and is admitted with a crisis. What treatment should the nurse anticipate? A. Give iron supplement. B. Encourage oral fluids. C. Apply cold compresses. D. Administer nonsteroidal anti-inflammatory drugs (NSAIDs).

B. Encourage oral fluids.

The nurse is caring for a client in skeletal traction. The nurse is assessing the pin sites and notes the presence of purulent drainage. Which nursing action would be most appropriate? A. Document the findings. B. Notify the health care provider. C. Apply antibiotic ointment to the pin sites. D. Clean the pin sites more frequently than prescribed.

B. Notify the health care provider.

The nurse is performing a skin assessment on a client and checks the client's skin for turgor. The nurse grasps a fold of the client's skin in which body area to best assess turgor? A. Sacral area B. Sternal area C. Top of the foot D. Back of the hand

B. Sternal area

The client is in active labor. She is 5 cm dilated and received 1 mg hydromorphone hydrochloride intravenously (IV) for labor pain 30 minutes ago. The nurse is assessing the response to the pain medication. Which response from the client demonstrates the normal effects of this medication at this time? A. "I feel numbness in my legs." B. "I feel almost no pain at all now." C. "I feel the pain still, but it is less." D. "I feel that my contractions have stopped."

C. "I feel the pain still, but it is less."

Human sexuality is interrelated with a variety of other nursing concepts that may affect sexuality or be affected by healthy sexual functioning. Prompt diagnosis and treatment of potential concerns related to concept overlap is an important nursing function. Which other concept is most likely to overlap with sexuality? A. Stress B. Gas exchange C. Pain D. Reproduction

D. Reproduction

The nurse is monitoring a newborn of a mother with diabetes mellitus. The nurse understands that the newborn is at risk for which complication? A. Hypercalcemia B. Hyperglycemia C. Hypobilirubinemia D. Respiratory distress syndrome

D. Respiratory distress syndrome

What is a priority nursing intervention to prevent hypothermia for a client at risk? A. Offer the client warm liquids such as tea. B. Provide the client with additional blankets. C. Close windows and doors to prevent drafts. D. Teach the client about ways to maintain warmth.

D. Teach the client about ways to maintain warmth

The nurse provides instructions to a client who is being discharged after undergoing a percutaneous renal biopsy. Which statement by the client indicates a need to reinforce the instructions? A. "A fever is normal following this procedure." B. "I should not work out at the gym for about 2 weeks." C. "I need to avoid any strenuous lifting for about 2 weeks." D. "I will call the health care provider if my urine becomes bloody."

A. "A fever is normal following this procedure."

A client with chronic hepatitis B says to the nurse, "My doctor prescribed adefovir for me, so I guess I'll soon be cured." Which nursing response would be therapeutic? A. "Although this medication cannot cure your disease, it will control it." B. "Yes. Although it's relatively new and its results remain tentative, it does seem so." C. "Yes, but just be certain to divide the dosage as the health care provider has indicated in the instructions." D. "Yes, it will cure the disease, but you will want to monitor for side effects and stop the medication immediately if any occur."

A. "Although this medication cannot cure your disease, it will control it."

The client reports to the emergency department with a 2-day history of abdominal cramping and watery diarrhea that has become bloody. What assessment question should the nurse ask related to the possibility of Escherichia coli? A. "Did anyone else eat the same food, and are they ill?" B. "Did you eat any improperly cooked eggs in the last 24 hours?" C. "Have you taken any over-the-counter medications in the last week?" D. "Do you have a family history of inflammatory bowel disease (IBD)?"

A. "Did anyone else eat the same food, and are they ill?"

A female client is suspected of having a vaginal infection caused by the organism Candida albicans. Which assessment question would elicit data associated with this infection? A. "Have you had any vaginal discharge?" B. "Do you have any blood in your urine?" C. "Have you noticed any swelling in your feet?" D. "Have you had any flank pain or headaches?"

A. "Have you had any vaginal discharge?"

The nurse is teaching a patient with a new diagnosis of systemic lupus erythematosus (SLE) about her disease. The nurse recognizes that the patient understands the information when making which statement? A. "I need to avoid getting infections because they will increase the immune response in my body, which can make my SLE worse." B. "I need to be sure to take all the available immunizations to keep me from getting sick." C. "Because of my SLE, my immune system is already diminished, so I need to avoid people with the flu." D. "As long as I take all my prescribed medications, I won't have to make any lifestyle changes as a result of my SLE."

A. "I need to avoid getting infections because they will increase the immune response in my body, which can make my SLE worse."

The nurse is teaching the family of an adolescent newly diagnosed with type 2 diabetes mellitus about the disorder. The parents verbalize understanding of the teaching if which statement is made? A. "I should begin to look for a fitness program for my child." B. "I'll give my child insulin as soon as I notice that he gets irritable and sweaty." C. "This condition was caused by the inability of the pancreas to produce insulin." D. "If I administer insulin as prescribed, my child will not suffer any long-term complications."

A. "I should begin to look for a fitness program for my child."

The nurse is assessing the client's family for understanding of the client's craniectomy, scheduled for the next day. Which statement best indicates the family understands this procedure? A. "Our father will have a piece of his skull removed to allow for the swelling on his brain to decrease." B. "Our father will have an opening in the head made by a drill to allow the fluid to be removed from his brain." C. "Our father will have the blood removed from his brain and be able to come home at the end of the day." D. "We will not be able to see our father for 48 hours because his surgery needs for him to remain in isolation."

A. "Our father will have a piece of his skull removed to allow for the swelling on his brain to decrease."

The nurse provides home care instructions to the parents of a 6-year-old child who has asymptomatic tuberculosis (TB) and is receiving pharmacotherapy. Which statement by the parents indicates a need for further teaching? A. "We will arrange for home schooling for the next 6 months." B. "We will be sure to have our child receive the influenza vaccination." C. "We will be careful to administer medications as prescribed to our child." D. "We will be sure to make the required follow-up appointments as scheduled."

A. "We will arrange for home schooling for the next 6 months."

At 0700, the nurse administers insulin lispro to a client with type 1 diabetes mellitus. What is the most important time period for the nurse to monitor the client for hypoglycemia? A. 0730 to 0930 B. 1000 to 1300 C. 1430 to 1730 D.1800 to 2100

A. 0730 to 0930

The nurse is assessing the extent of pitting edema in a client with heart failure. The nurse gently presses a finger on the client's ankle and notes a barely perceptible pitting. The nurse interprets this finding as which measurement of pitting edema? A. 1+ B. 2+ C. 3+ D. 4+

A. 1+

The embryonic period is critical because external and internal structures in the fetus are forming. All teratogens should be avoided from A. 4 to 8 weeks. B. 8 to 12 weeks. C. 12 to 16 weeks. D. 16 to 20 weeks.

A. 4 to 8 weeks.

A child newly diagnosed with type 1 diabetes mellitus who is receiving insulin suddenly experiences signs of a hypoglycemic reaction. Which item should the nurse give to the child immediately? A. 8 oz (225 mL) of skim milk B. ½ cup of diet cola C. 1 teaspoon of honey D. 1 teaspoon of sugar

A. 8 oz (225 mL) of skim milk

Select all the organs from the following list that are part of the immune system. A. Adenoids B. Appendix C. Bone marrow D. Gallbladder E. Liver F. Thyroid gland

A. Adenoids B. Appendix C. Bone marrow

A client has been admitted to the acute care unit with a diagnosis of thyroid storm. What is the priority nursing intervention? A. Administer antipyretic medications, as prescribed. B. Identify stressors that may have triggered the condition. C. Begin a continuous infusion of normal saline 0.9% at keep vein open (KVO) rate. D. Obtain a blood specimen to evaluate levels of thyroid-stimulating hormone (TSH), T3, and T4.

A. Administer antipyretic medications, as prescribed.

The nurse looks at a telephone laboratory report and notes that a client's serum potassium level is 5.5 mEq/L (5.5 mmol/L). Based on this laboratory result, the nurse should take which most appropriate action? A. Contact the health care provider. B. Administer the prescribed potassium chloride. C. Instruct the client to include high-potassium foods in the diet. D. Call the dietary department and request that a banana or a glass of orange juice be delivered to the client daily.

A. Contact the health care provider.

The nurse is reviewing the laboratory results of a client and notes that the serum sodium level is 150 mEq/L (150 mmol/L). The nurse understands that this laboratory result would be noted in a client who had which condition? A. Dehydration B. Addison's disease C. A severe burn injury D. Adrenal insufficiency

A. Dehydration

The nurse is assessing the client after having a fractured hip repaired. The client is exhibiting symptoms of confusion, which came on rapidly after surgery. The client is also exhibiting hallucinations. Which syndrome is this client most likely experiencing? A. Delirium B. Dementia C. Depression D. Alzheimer's disease

A. Delirium

The client presents with these findings: constipation, polyuria, tachycardia, and serum osmolality of 325 mOsm/kg (325 mmol/kg) and urine specific gravity of 1.000. The nurse would be correct to suspect which disorder? A. Diabetes insipidus B. Hyperparathyroidism C. Hyperglycemic-hyperosmolar state (HHS) D. Syndrome of inappropriate antidiuretic hormone (SIADH)

A. Diabetes insipidus

The nurse is performing pin care to the pin sites of a client in skeletal traction and notes the presence of serous drainage. The nurse should take which nursing action? A. Document the findings. B. Notify the health care provider. C. Remove 2 pounds of weight from the traction. D. Lift the weights and place them on the bed so that the health care provider can assess the client.

A. Document the findings.

The nurse is preparing to insert a peripheral intravenous catheter into a client. What areas on the client does the nurse know to avoid? Select all that apply. A. Edematous extremity B. A nonparalyzed arm C. A skin area that is infected D. The arm on the same side as a mastectomy E. An arm that has an arteriovenous fistula/shunt for dialysis F. An arm with a peripherally inserted intravenous catheter already in place

A. Edematous extremity C. A skin area that is infected D. The arm on the same side as a mastectomy E. An arm that has an arteriovenous fistula/shunt for dialysis

The teaching plan for a child with tinea corporis (ringworm) who is on the school wrestling team should include which measures? Select all that apply. A. Perform frequent skin checks. B. Apply antibiotic ointment for 3 full days. C. Shower immediately after practices with soap and water. D. Avoid any treatment for areas close to the eyes or in the scalp area. E. Stop taking griseofulvin as soon as lesions show improvement. F. Avoid contact with other wrestlers until lesions take on a honey-colored, crusted appearance.

A. Perform frequent skin checks. C. Shower immediately after practices with soap and water.

A child presents to the emergency department with complaints of fever, a petechial rash, nausea, vomiting, headache, and nuchal rigidity. What is the triage nurse's priority action? A. Place the child on isolation precautions. B. Administer acetaminophen for fever. C. Administer the meningococcal conjugate vaccination. D. Draw blood for a prescribed white blood cell (WBC) count.

A. Place the child on isolation precautions.

The nurse should take which actions when caring for a child experiencing a seizure? Select all that apply. A. Place the child on the side. B. Restrain the child to prevent head trauma. C. Ease the child who is sitting in a wheelchair onto the floor. D. Pull the teeth apart and place an oral airway into the child's mouth. E. Offer the child a sip of water to elicit the swallowing reflex and prevent aspiration.

A. Place the child on the side. C. Ease the child who is sitting in a wheelchair onto the floor.

What manifestations would an older adult with depression exhibit? Select all that apply. A. Poor appetite B. Lack of energy C. Early morning insomnia D. Excessive daytime sleeping E. Starting to drink alcohol daily F. Unwillingness to participate in social activities

A. Poor appetite B. Lack of energy C. Early morning insomnia D. Excessive daytime sleeping F. Unwillingness to participate in social activities

The nurse is assessing a client who is exhibiting signs of autonomic dysreflexia. What is the priority action the nurse must take for this client? A. Raise the head of the bed. B. Obtain an oxygen saturation. C. Document the occurrence, treatment, and response. D. Insert a Foley catheter, per as-needed (prn) prescription.

A. Raise the head of the bed.

The client states he is having difficulty keeping his blood glucose under control with his usual dosage of insulin. The nurse should assess what factors to evaluate possible reasons for this recent change? Select all that apply. A. Recent history of illness B. Reduced carbohydrate intake C. Calibration of the glucometer D. Location of insulin administration E. Increase in the intensity of exercise

A. Recent history of illness C. Calibration of the glucometer D. Location of insulin administration

Which assessment finding should the nurse expect to note in a child with hypovolemic shock? A. Reduced urinary output B. Elevated blood pressure C. Brisk capillary refill time D. Elevated central venous pressure (CVP)

A. Reduced urinary output

The client with a spinal cord injury at level C4 is brought to the emergency department. What is the priority nursing assessment? A. Respiratory effort B. Rectal temperature C. Level of intact sensation D. Glasgow Coma Scale (GCS) score

A. Respiratory effort

The nurse is caring for a child who sustained a head injury from a fall. The nurse should perform which actions in the care of the child? Select all that apply. A. Restrict oral fluid intake. B. Elevate the head of the bed. C. Perform neurological assessments. D. Encourage coughing and deep breathing. E. Place the child in a flat position during sleep.

A. Restrict oral fluid intake. B. Elevate the head of the bed. C. Perform neurological assessments.

A school-aged child has contracted head lice at school. The nurse should include which instruction about treatment for head lice? A. Seal nonwashable items in plastic bags for 14 days. B. Use a soft bristled hairbrush to rid the scalp of nits. C. Retreatment is not necessary and can cause scalp irritation. D. There is no need to wash bed linens until after the first 48 hours of treatment.

A. Seal nonwashable items in plastic bags for 14 days.

The client is a 20-year-old woman who is taking combined oral contraception pills for birth control. She has been on the pill for 2 months. She calls the clinic and tells the nurse that she is spotting on and off between her periods. What would be the most appropriate information for this client? A. Spotting is common in the first months of pill use. B. She should stop the pills and come in for evaluation. C. This pattern of bleeding indicates that the pill is ineffective. D. She should try taking the pills in the morning rather than in the evening.

A. Spotting is common in the first months of pill use.

A client is taking a thyroid hormone replacement medication. The nurse knows that thyroid hormones have which effects? Select all that apply. A. Stimulate the heart. B. Promote bradycardia. C. Encourage energy use. D. Reduce heat production. E. Promote growth and development.

A. Stimulate the heart. C. Encourage energy use. E. Promote growth and development.

During morning hand-off report, the nurse notes that the client has been diagnosed with bacterial conjunctivitis. Which of these are signs and symptoms of bacterial conjunctivitis? Select all that apply. A. Tearing B. Photophobia C. Mild edema D. Burning sensation E. Blood vessel dilation F. Thick mucous discharge

A. Tearing C. Mild edema E. Blood vessel dilation F. Thick mucous discharge

The labor room nurse is preparing to give report to the postpartum nurse about a client who had a normal vaginal birth complicated by postpartum hemorrhage. Which data are most relevant to the client's care at this time to include in the report to the postpartum nurse? Select all that apply. A. The client's pulse is 112 beats/min. B. The client has not yet voided since delivery. C. The client had a 30-pound weight gain in pregnancy. D. The client tested positive for group B Streptococcus (GBS) during pregnancy. E. The client received methylergonovine 0.2 mg intramuscularly, 1 hour ago.

A. The client's pulse is 112 beats/min. B. The client has not yet voided since delivery. E. The client received methylergonovine 0.2 mg intramuscularly, 1 hour ago.

The client gave birth yesterday, and the nurse is planning for her discharge today. Which data indicate the need for further nursing assessment before discharge? A. The maternal blood type is A negative. B. The maternal rubella titer shows immunity. C. The maternal hemoglobin is 11.6 g/dL postpartum. D. The mother received the pertussis vaccine at 34 weeks' pregnancy.

A. The maternal blood type is A negative.

The client is a 30-year-old who had a previous cesarean section (C/S) and would like to have a vaginal birth after cesarean section (VBAC) with the current pregnancy. Which data would indicate increased risk of uterine rupture in labor? A. The uterine incision was vertical. B. The previous C/S was 3 years ago. C. The previous infant weighed 7 pounds. D. The reason for the previous C/S was breech presentation.

A. The uterine incision was vertical.

The client is an elementary school teacher. She is at 29 weeks' pregnancy and expresses concern about a recent outbreak of whooping cough or pertussis among children at her school. The client has never been vaccinated. What can the nurse tell her about the whooping cough or pertussis vaccine? A. The vaccine should be administered to the client now. B. The vaccine should be administered to the client postpartum. C. The vaccine should be administered in the first trimester of pregnancy. D. The vaccine is not recommended for use in pregnancy or during lactation.

A. The vaccine should be administered to the client now.

A client is admitted to the emergency department exhibiting bizarre and violent behavior. The health care provider suspects that the client has ingested hallucinogens. What physiologic effects would the client exhibit? Select all that apply. A. Tremor B. Hypothermia C. Incoordination D. Hyperreflexia E. Muscular weakness

A. Tremor C. Incoordination D. Hyperreflexia E. Muscular weakness

The nurse is developing the plan of care for the client who had a stroke. This client had a parietal lobe infarct, resulting in a spatial perceptual loss. Which problem is most appropriate to address for this client? A. Unilateral neglect B. Feelings of inadequacy C. Inability to move around D. Inability to expectorate secretions

A. Unilateral neglect

On assessment, the nurse notes that the client's deep tendon reflexes are depressed. This finding can occur in which conditions? Select all that apply. A. Use of opioids B. Heavy sedation C. Caffeine intake D. Use of anorexiants E. Psychogenic disorder

A. Use of opioids B. Heavy sedation

The nurse is teaching a new diabetic client and the family of the client about the disorder. What steps should the nurse include in the teaching process? Select all that apply. A. Use the teach-back method or return demonstration when appropriate. B. Spend time teaching clients who are showing signs of compliance only. C. Document the teaching and what was taught during the teaching session. D. Document who was present and evaluation of understanding of the teaching. E. Explain to the client what could happen if information shared during teaching is not followed. F. Determine what it means to the client and family to have diabetes in a language they understand.

A. Use the teach-back method or return demonstration when appropriate. C. Document the teaching and what was taught during the teaching session. D. Document who was present and evaluation of understanding of the teaching. E. Explain to the client what could happen if information shared during teaching is not followed. F. Determine what it means to the client and family to have diabetes in a language they understand.

The client is prescribed neurological checks hourly after a craniotomy. At 0100, the client reports a headache. The nurse obtains a Glasgow Coma Scale (GCS) score of 14 and administers acetaminophen as needed. At 0200, the client appears to be sleeping. What action should the nurse take? A. Wake the client and ask what day it is. B. Let the client sleep but verify respiratory rate. C. Document "relief obtained" and recheck at 0300. D. Wake the client and check for the presence of ecchymosis.

A. Wake the client and ask what day it is.

Which teaching point is a priority for the parents of a toddler with leukemia to include in the daily care plan? A. Wash hands frequently. B. Provide healthy food choices. C. Provide daily exercise and play time. D. Keep all health care provider appointments.

A. Wash hands frequently.

A patient admitted to an acute care floor has rubor of an area of injury on the left lower extremity. The nurse understands that this redness is caused by A. vasodilation. B. extravasation. C. neutrophils. D. exudate.

A. vasodilation

A client is tested for human immunodeficiency virus (HIV) with an enzyme-linked immunosorbent assay (ELISA) test, and the test result is positive. The client is very upset and asks the nurse if this means that he definitely has HIV. How should the nurse respond to the client? A. "Yes, you definitely have HIV." B. "Another test will be done to determine if you have HIV." C. "False-positive results are reported all of the time and you should not be worried." D. "A positive test means that the infection was diagnosed early in the initial infection period."

B. "Another test will be done to determine if you have HIV."

The wife of a man who sustained an eye injury calls the emergency department and speaks to the nurse. The wife reports that her husband was hit in the eye area by a piece of board while building a shed in the backyard. The nurse should advise the wife to take which immediate action? A. Call an ambulance. B. Apply ice to the affected eye. C. Irrigate the eye with cool water. D. Bring the husband to the emergency department.

B. Apply ice to the affected eye.

A client diagnosed with schizophrenia is demonstrating the classic behaviors associated with a psychotic relapse. How should the nurse best assess the client's current potential to be violent? A. Arrange for one-on-one observation of the client. B. Ask the client, "What are your voices saying to you?" C. Ask family members if the client is generally an angry, aggressive person. D. Review the client's medical record to determine when the behaviors began.

B. Ask the client, "What are your voices saying to you?"

According to the DSM-5 diagnostic criteria for schizophrenia, which are characteristic negative symptoms of schizophrenia? Select all that apply. A. Paranoia B. Blunted affect C. Poor judgment D. Hallucinations E. Social withdrawal F. Disordered thinking

B. Blunted affect C. Poor judgment E. Social withdrawal

The nurse notes that a client's serum potassium level is 5.8 mEq/L (5.8 mmol/L). The nurse interprets that this is an expected finding in the client with which problem? A. Diarrhea B. Burn injury C. Diabetes insipidus D. Pulmonary edema being treated with loop diuretics

B. Burn injury

The nurse instructs a client taking a potassium-retaining diuretic about foods high in potassium that need to be avoided. The nurse determines that the client needs further instruction if the client states that which food is high in potassium? A. Kiwi B. Celery C. Oranges D. Dried fruit

B. Celery

The home care nurse is assessing a client who began using peritoneal dialysis 1 week ago. The nurse should suspect the onset of peritonitis if which finding is noted on assessment? A. Anorexia B. Cloudy dialysate output C. Mild abdominal discomfort D. Oral temperature of 99.0° F (37° C)

B. Cloudy dialysate output

The home care nurse is assessing a client who began using peritoneal dialysis 1 week ago. The nurse should suspect the onset of peritonitis if which finding is noted on assessment? A. Anorexia B. Cloudy dialysate output C. Mild abdominal discomfort D. Oral temperature of 99.0° F (37° C)

B. Cloudy dialysate output

The nurse is assessing a client admitted with a medical diagnosis of subdural hematoma. Which assessment requires further action by the nurse? A. Glasgow Coma Scale (GCS) score of 15 B. Complaint of blurry vision C. Blood pressure of 130/80 mm Hg D. Complaint of continuing headache

B. Complaint of blurry vision

The nurse is observing a student donning a pair of sterile gloves and preparing a sterile field. The nurse should intervene if the student performs which actions? Select all that apply. A. Puts the right glove on and then the left glove B. Dons the sterile gloves without washing the hands C. Uses the inner wrapper of the gloves as a sterile field D. Maintains the gloved hands below the level of the waist E. Touches the gloves on the overbed table, removes them, and dons another sterile pair

B. Dons the sterile gloves without washing the hands D. Maintains the gloved hands below the level of the waist

The client is a 22-year-old gravida 1 at 39 weeks' pregnancy in active labor. She has stated that she prefers natural childbirth and does not want an epidural or opioid medications. She has been coping with labor by deep breathing and using differing positions, including walking, sitting on a birth ball, and positioning on her hands and knees. She states she feels tired and that the pain is now much more intense. The nurse midwife examines her and finds her to be 6 cm dilated or 100% effaced with the fetal head at station 0. What should the nurse do at this time to help the client cope with the pain of labor? A. Encourage her to keep walking. B. Encourage water immersion in a tub of warm water. C. Encourage the client to accept an epidural because she may still have a long way to go in labor. D. Encourage her to ask the nurse midwife to perform artificial rupture of the membranes to enhance labor progress.

B. Encourage water immersion in a tub of warm water.

The nurse is caring for a hospitalized client with pharyngitis. The client had been responding well to interventions but has now developed restlessness, stridor, and decreased oxygen saturation. The nurse recognizes this as an emergency situation most likely attributed to what development? A. Sinusitis B. Epiglottitis C. Pneumonia D. Rheumatic fever

B. Epiglottitis

The nurse manager of a neuro-medical surgical unit reviewing potential manifestations of seizures with an orientee would become concerned if the new nurse included which of the following dysfunctions as a manifestation? A. Autonomic B. Family C. Motor D. Sensory

B. Family

The home care nurse is visiting a postpartum client. The nurse reviews the information in the client's medical record and performs an assessment on the client. The nurse should suspect endometritis if which finding is noted? A. Breast engorgement B. Fever beginning 3 days postpartum C. Slightly elevated white blood cell count D. Lochia rubra on the second day postpartum

B. Fever beginning 3 days postpartum

The nurse is completing an admission assessment of a new patient to the unit. The nurse notes a long, thin, fading scar on the patient's abdomen in the right lower quadrant. What is the best explanation for the scar's appearance? A. Optimal functioning of the inflammatory process after an injury B. Fibrous tissue replacing damaged tissue when injury is extensive C. The development of chronic inflammation D. A surgical incision

B. Fibrous tissue replacing damaged tissue when injury is extensive

The client is a 42-year-old gravida 8, para 8 who had a vaginal delivery of a 9-pound 4-ounce infant 30 minutes ago. The nurse is assessing the client and notes a large amount of vaginal bleeding and blood clots. What is the first action the nurse should take? A. Check the client's blood pressure. B. Firmly massage the uterine fundus. C. Assist the client up to the bathroom to void. D. Clean the perineal area of blood, and provide clean pads.

B. Firmly massage the uterine fundus.

The nurse is assigned to care for a client with an abdominal wound infection who is on contact precautions. The nurse gathers supplies before entering the client's room and should obtain which necessary protective items? A. Gloves and a mask B. Gloves and a gown C. Gloves and goggles D. Gloves and a face shield

B. Gloves and a gown

The client is at 6 weeks' pregnancy and has called the clinic to speak to the nurse. The client complains of light vaginal bleeding and new onset of severe left lower quadrant pain. What should the nurse advise the client to do? A. Maintain bed rest, and abstain from sexual intercourse. B. Go to the emergency department to be evaluated immediately. C. Make an appointment to see the nurse midwife within 2 weeks. D. Increase fluids and fibers in the diet, and call back if symptoms continue.

B. Go to the emergency department to be evaluated immediately.

A client newly diagnosed with type 1 diabetes mellitus is taking an intermediate-acting insulin at 0700 daily. The nurse should monitor the client closely for which signs and symptoms in the late afternoon? A. Increased appetite and abdominal pain B. Hunger, shakiness, and cool, clammy skin C. Thirst, red dry skin, and fruity breath odor D. Increased urination and rapid deep breathing

B. Hunger, shakiness, and cool, clammy skin

A client is admitted with the symptoms of dry, flushed skin; confusion; and postural hypotension. Which laboratory result should the nurse most likely expect to note? A. Hyperkalemia B. Hyponatremia C. Hypercalcemia D. Hypermagnesemia

B. Hyponatremia

The nurse is preparing to care for a pediatric client with an intravenous solution infusing. The nurse should ensure that which item is in place to prevent fluid overload in this client? A. Armboard B. Infusion pump C. Macrodrip infusion set D. Large-bore intravenous catheter

B. Infusion pump

A client diagnosed with acquired immunodeficiency syndrome (AIDS) is hospitalized. The nurse develops a plan of care and determines that which intervention is the priority? A. Providing emotional support to the client B. Instituting measures to prevent infection in the client C. Identifying the ways that AIDS can be contracted by others D. Discussing the ways that the client contracted the AIDS virus

B. Instituting measures to prevent infection in the client

A client arrives at the prenatal clinic for the first prenatal assessment. She tells the nurse that the first day of her last menstrual period (LMP) was August 19, 2016. Using Nägele's rule, the nurse determines the estimated date of delivery as which date? A. May 12, 2017 B. May 26, 2017 C. June 12, 2017 D. June 26, 2017

B. May 26, 2017

The client has had a nasogastric tube attached to low continuous suction for 2 days. An arterial blood gas value is drawn, and the results are as follows: pH 7.48, Paco2 40 mm Hg, and HCO3 30 mEq/L (30 mmol/L). The nurse recognizes that the client is experiencing which acid-base imbalance? A. Metabolic acidosis B. Metabolic alkalosis C. Respiratory acidosis D. Respiratory alkalosis

B. Metabolic alkalosis

A client has a diagnosis of hyperphosphatemia. The nurse should tell the client to avoid consuming which food item? A. Tea B. Milk C. Coffee D. Grape juice

B. Milk

The nurse is caring for a child who experienced significant blood loss from surgery. What is the nurse's priority action when caring for this child? A. Monitor the child's temperature and white blood cell (WBC) count. B. Monitor the child's intake and output (I & O) along with blood pressure. C. Review the child's most recent urinalysis and basic metabolic panel results. D. Encourage the child's parents and siblings to stay with the child as much as possible.

B. Monitor the child's intake and output (I & O) along with blood pressure.

The nurse has been assigned to care for a client who is positive for hepatitis C. Which occurrence places the nurse at risk for transmission of the disease? A. Contact with feces B. Needle-stick injury C. Symptomatic client D. Being coughed on by the client

B. Needle-stick injury

The nurse is reviewing the laboratory results of a client with bladder cancer and bone metastasis and notes that the calcium level is 15 mg/dL (3.75 mmol/L). Which nursing action is most appropriate? A. Document the findings. B. Notify the health care provider. C. File the report in the client's record. D. Increase calcium-containing foods in the diet.

B. Notify the health care provider.

The nurse notes redness, warmth, and a purulent drainage at the insertion site of a central venous catheter in a client receiving total parenteral nutrition (TPN). The nurse should take which priority action? A. Change the intravenous tubing. B. Notify the health care provider. C. Slow the rate of infusion of the TPN. D. Call the pharmacy for a new bag of TPN solution.

B. Notify the health care provider.

While performing a neurological assessment on the client's eye movement, the eye jerks while looking to the right. How should the nurse correctly document this finding? A. Ptosis B. Nystagmus C. Accommodation D. Disconjugate gaze

B. Nystagmus

What teaching should the nurse provide to an older client that would be most effective in preventing shingles (herpes zoster, varicella-zoster)? A. Maintain enteric precautions. B. Obtain the herpes zoster vaccine. C. Perform frequent hand washing. D. Wear a condom during sexual activity.

B. Obtain the herpes zoster vaccine.

Norfloxacin is prescribed for a client with acute prostatitis. The nurse should instruct the client to take the medication at what time? A. With meals B. One hour before meals C. At bedtime with a snack D. With a snack in the late afternoon

B. One hour before meals

The client with acute pancreatitis presents to the emergency department. What is the priority assessment? A. Pain rating B. Orthostatic vital signs C. Recent alcohol intake D. Presence of cola-colored urine

B. Orthostatic vital signs

A 9-year-old child diagnosed with leukemia has been prescribed cyclophosphamide. The nurse should assess for which adverse effect related to this medication? A. Staggering gait B. Pain on urination C. Heart palpitations D. Sensitivity to light

B. Pain on urination

Before measuring a client's oral temperature using an electronic device, what should the nurse do first? A. Don sterile gloves. B. Perform hand hygiene for at least 15 to 20 seconds. C. Clean the temperature probe cover with an alcohol swab. D. Apply a plastic probe cover to the thermometer probe stem.

B. Perform hand hygiene for at least 15 to 20 seconds.

A client at 10 weeks' gestation has type 1 diabetes mellitus and is receiving prenatal care at a high-risk clinic. The nurse teaches the client about the early signs of hyperglycemia and should tell the client that which is an early sign? A. Hunger B. Polyuria C. Shakiness D. Nervousness

B. Polyuria

The health care provider prescribes diphenhydramine to be administered to a client before a blood transfusion. The nurse should tell the client that this medication has been prescribed to have which therapeutic effect? A. Prevent chills and a fever. B. Prevent an urticarial reaction. C. Assist in the absorption of the blood product. D. Promote movement of the red blood cells into the bone marrow.

B. Prevent an urticarial reaction.

A client who is recovering from a stroke (brain attack) has residual dysphagia. To assist in assessing the client's swallowing ability the nurse should ask the client to perform which action? A. Swallow some water. B. Produce an audible cough. C. Suck on a piece of hard candy. D. Swallow a teaspoon of applesauce.

B. Produce an audible cough.

The nurse is working on a mental health unit and administering many antipsychotic medications to clients. What are some indications for administering these medications? Select all that apply. A. Anxiety B. Schizophrenia C. Bipolar disorders D. Delusional disorders E. Depressive psychoses F. Medication-induced psychoses

B. Schizophrenia C. Bipolar disorders D. Delusional disorders E. Depressive psychoses F. Medication-induced psychoses

The client has a prepregnancy body mass index (BMI) of 28 and comes to the clinic at 6 weeks' pregnancy. How should the nurse advise her regarding her total weight gain in pregnancy? A. She should be advised to gain 11 to 20 pounds. B. She should be advised to gain 15 to 25 pounds. C. She should be advised to gain 25 to 35 pounds. D. She should be advised to gain 28 to 40 pounds.

B. She should be advised to gain 15 to 25 pounds.

A 12-year-old with diabetes mellitus enters the emergency department complaining of extreme thirst and weakness with a respiratory rate of 32 breaths/min. The child's skin is warm and dry, and his mother states he has been urinating very frequently. What is the nurse's priority action? A. Apply a cooling blanket. B. Start a peripheral intravenous (IV) line. C. Instruct the child to breathe into a paper bag. D. Encourage the child to drink the prescribed electrolyte and glucose solution.

B. Start a peripheral intravenous (IV) line.

The client is a 17-year-old gravida 2 para 2. She is successfully breastfeeding her newborn. She is preparing for discharge with her newborn. Her boyfriend and their 1-year-old are accompanying her. Which statement by the client indicates the need for further teaching? A. "I plan to receive the birth control shot." B. "I plan to have progestin implant inserted in my arm." C. "I plan to begin using birth control when I'm done breastfeeding." D. "I plan to use the 10-year intrauterine device (IUD) because I can't handle any more kids right now."

C. "I plan to begin using birth control when I'm done breastfeeding."

A client has been prescribed clozapine after other atypical antipsychotic medications failed to produce acceptable management of schizophrenic-related symptomology. At what point should the nurse alert the client's health care provider to the possible need to adjust the client's medication therapy plan? A. When the client begins to demonstrate fine facial tremors B. When the client reports having a sore throat and mouth sores C. When the client finds it difficult to sleep at night but naps during the day D. When the client demonstrates paranoid ideations regarding certain staff members

B. When the client reports having a sore throat and mouth sores

A nurse is assessing a menopausal female and discussing sexuality. Which statement is accurate regarding physiological effects of menopause on sexual health? A. Decreased lubrication is frequently cited as the cause for sexual problems. B. Women who have undergone hysterectomy no longer desire to be sexually active. C. Hot flashes are often bothersome and lead to decreased sexual interest. D. Women taking hormone replacement therapy may not experience climax during sex.

B. Women who have undergone hysterectomy no longer desire to be sexually active.

The nurse is working with a nursing student in educating a newly diagnosed client with diabetes mellitus about metformin. Which statement by the nursing student, if made to the client, requires intervention by the nurse? A. "The side effects of metformin should decrease over time." B. "You should take metformin with food to decrease stomach upset." C. "If you take high doses of metformin, you will definitely develop hypoglycemia." D. "If you need a computed tomography (CT) scan, let them know that you take metformin."

C. "If you take high doses of metformin, you will definitely develop hypoglycemia."

The client is prescribed an intranasal glucocorticoid spray. Which statement should the nurse include when providing education about this medication? A. "You should rinse your mouth out each time you use this medication." B. "This decongestant constricts blood vessels to shrink swollen nasal passages." C. "It may take a couple of weeks or more before you experience significant benefit." D. "Because it is administered directly to the affected area, there are no side effects."

C. "It may take a couple of weeks or more before you experience significant benefit."

A client states that her blood glucose at the health fair she attended yesterday was 145 mg/dL (8.1 mmol/L), and she is concerned that she will need to begin medication for diabetes mellitus. What is the best response by the nurse? A. "I am sure everything is fine; you are not overweight." B. "Is there a history of diabetes in your immediate family?" C. "Unless you were fasting for this test, the result does not mean you have diabetes." D. "I would recommend asking your health care provider about starting an oral hypoglycemic medication."

C. "Unless you were fasting for this test, the result does not mean you have diabetes."

The nurse is in the emergency department is performing an assessment on a client who sustained a right finger laceration from a fish hook while fishing. The nurse should ask the client which PRIORITY question? A. "When was your last physical Examination?" B. "Have you had a chest x-ray in the last year?" C. "When did you receive your last tetanus immunization?" D. "Have you ever sustained this type of injury int he past?"

C. "When did you receive your last tetanus immunization?"

A client with severe hyponatremia is being treated with intravenous hypertonic saline (3%). The nurse determines that the treatment is effective when the laboratory results reveal which sodium level? A. 120 mEq/L (120 mmol/L) B. 130 mEq/L (130 mmol/L) C. 140 mEq/L (140 mmol/L) D. 150 mEq/L (150 mmol/L)

C. 140 mEq/L (140 mmol/L)

The nurse should assess for which clinical manifestations in a client with hypothyroidism? A. Goiter, diarrhea, and hoarseness B. Anxiety, palpitations, and hair loss C. Constipation, anemia, and periorbital edema D. Fatigue, nausea, and a leg ulcer that will not heal

C. Constipation, anemia, and periorbital edema

A client begins taking a sulfonylurea once daily. The nurse should observe for which intended effect of this type of medication? A. Weight loss B. Resolution of infection C. Decreased blood glucose D. Decreased blood pressure

C. Decreased blood glucose

Which finding indicates to the nurse that vasopressin is effectively managing symptoms associated with diabetes insipidus? A. Polydipsia B. Hypotension C. Decreased urine output D. Increased serum osmolality

C. Decreased urine output

The nurse is caring for a child undergoing peritoneal dialysis for chronic kidney disease. The health care provider (HCP) should be alerted immediately about which assessment finding? A. Serum potassium level is 4.6 mEq/L (4.6 mmol/L). B. Arteriovenous (AV) fistula has a palpable thrill. C. Dialysate solution draining from the child becomes cloudy. D. Delayed growth is evident on the child's growth curve chart.

C. Dialysate solution draining from the child becomes cloudy.

The nurse is monitoring a client with hyperparathyroidism for signs of hypercalcemia. The nurse would expect to note which finding if hypercalcemia is present? A. Paresthesias B. Positive Chvostek's sign C. Diminished bowel sounds D. Hyperactive deep tendon reflexes

C. Diminished bowel sounds

A child is being assessed to determine whether the child is a victim of Munchausen syndrome by proxy (MSBP). Which questions should be considered by the nurse? Select all that apply. A. Does the child appear to fear the caregiver? B. Does the child appear to have suffered sexual abuse? C. Does diagnostic evidence support the reported history? D. Has anyone but the caregiver witnessed the symptoms? E. Is the child's condition consistent with the reported history? F. Is treatment being provided primarily because of the caregiver's demands?

C. Does diagnostic evidence support the reported history? D. Has anyone but the caregiver witnessed the symptoms? E. Is the child's condition consistent with the reported history? F. Is treatment being provided primarily because of the caregiver's demands?

The emergency department nurse is caring for a client who was in a motor vehicle crash and is told by the paramedics that the client hit a tree, was not wearing a seatbelt, and his head hit the windshield. What is a priority action for the nurse? A. Insert a second intravenous (IV) line. B. Assess for the presence of Doll's eyes. C. Leave on the hard immobilization collar. D. Obtain a Glasgow Coma Scale (GCS) score.

C. Leave on the hard immobilization collar.

When a client is prescribed the atypical antipsychotic medication olanzapine, the nurse will best demonstrate an understanding of its possible side effects by taking which action? A. Monitor urinary output. B. Monitor apical heart rate. C. Monitor glucose levels by fingerstick. D. Monitor peripheral pulse quality and rates.

C. Monitor glucose levels by fingerstick.

Intracranial function can be disrupted by which degenerative disease of the brain? A. Encephalitis B. Meningitis C. Parkinson's disease D. Brain tumors

C. Parkinson's disease

The nurse reviews a nursing plan of care developed by a nursing student for a client with a brain attack (stroke) who has dysphagia. The nurse determines that the student needs to further research interventions for the care of a client with dysphagia if which intervention is noted in the plan? A. Thicken liquids. B. Assess for the presence of a swallow reflex. C. Place the food on the affected side of the mouth. D. Provide ample time for the client to chew and swallow.

C. Place the food on the affected side of the mouth.

The mother of a teenage son who has been engaging in exhibitionism asks, "Is there a chance that he will ever stop this behavior?" Based on the nurse's understanding of this form of paraphilia, what is the best response to this question? A. This form of paraphilia is always a precursor to rape B. This form of paraphilia always progresses into pedophilia. C. This form of paraphilia appears to resolve as the male ages. D. This form of paraphilia is always required for arousal to occur.

C. This form of paraphilia appears to resolve as the male ages.

The mother of a teenage son who has been engaging in exhibitionism asks, "Is there a chance that he will ever stop this behavior?" Based on the nurse's understanding of this form of paraphilia, what is the best response to this question? A. This form of paraphilia is always a precursor to rape. B. This form of paraphilia always progresses into pedophilia. C. This form of paraphilia appears to resolve as the male ages. D. This form of paraphilia is always required for arousal to occur.

C. This form of paraphilia appears to resolve as the male ages.

Which is the best reason for inserting an indwelling urinary catheter? A. To obtain a sterile urine specimen B. To measure urinary output in all hospitalized clients C. To prevent bladder distention during abdominal surgery D. To decrease the amount of times bed linens need to be changed

C. To prevent bladder distention during abdominal surgery

The client with full-thickness burns over 50% total body surface is brought to the emergency department. Rapid infusion of intravenous normal saline is started. What is the best indication that the client has adequate hydration? A. Presence of tears B. Moist mucous membranes C. Urine output of 40 mL/hr D. Capillary refill less than 2 seconds

C. Urine output of 40 mL/hr

The nurse enters the room of a client with diabetes mellitus and finds the client difficult to arouse. The client's skin is cool and clammy, and the client's pulse rate is elevated from the client's baseline. The nurse immediately implements which action? A. give the client a glass of orange juice B. prepare an intravenous (IV) insulin solution C. check the client's capillary blood glucose D. administer and IV bolus dose of 50% dextrose

C. check the client's capillary blood glucose

A mother tells the clinic nurse that she does not want her child to receive any immunizations because she has heard that they cause serious illnesses. The nurse should make which appropriate statement to the mother? A. "Are you afraid your child is going to die from the injection?" B. "Why are you afraid? Children are immunized every day without a problem." C. "There will be a slight discomfort at the time of the injection, but that is all that will happen." D. "I can see you are very concerned about your child. What do you think might happen after an immunization is given?"

D. "I can see you are very concerned about your child. What do you think might happen after an immunization is given?"

The client is diagnosed with hepatitis B. What question would be most helpful to determine how the disease was contracted? A. "Did you eat any raw shellfish?" B. "How much alcohol do you drink?" C. "Have you donated blood recently?" D. "Do you use intravenous (IV) drugs?"

D. "Do you use intravenous (IV) drugs?"

The nurse is providing care to four clients. When is it best for the nurse to plan on using soap and water for hand hygiene rather than an antiseptic hand rub? A. After repositioning a client who is comatose B. After taking a blood pressure on a client with hypertension C. After administering intramuscular pain medicine for the client with kidney stones D. After being splashed on the hands with sputum from a client with pneumonia

D. After being splashed on the hands with sputum from a client with pneumonia

A client is scheduled for a diagnostic procedure requiring the injection of a radiopaque dye. The nurse should check which most important information before the procedure? A. Intake and output B. Height and weight C. Baseline vital signs D. Allergy to iodine or shellfish

D. Allergy to iodine or shellfish

A married couple present to the preconceptual clinic with questions about how a fetus's chromosomal sex is established. What is the nurse's best response? A. At ovulation, chromosomal sex is established. B. At ejaculation, chromosomal sex is established. C. At climax, chromosomal sex is established. D. At fertilization, chromosomal sex is established.

D. At fertilization, chromosomal sex is established.

The client had a cesarean section under spinal anesthesia 22 hours ago. She is now complaining of a severe headache. The headache becomes worse when she is upright and disappears when she is supine. The health care provider plans to administer a blood patch. How should the nurse explain this procedure to the client? A. A blood patch is applied to the lower back of the client. B. Blood is mixed with opioids to relieve the severe headache. C. A blood patch is placed in the head to relieve the headache. D. Blood is injected into the epidural space to seal a leak of cerebrospinal fluid.

D. Blood is injected into the epidural space to seal a leak of cerebrospinal fluid.

The nurse is caring for a client with a T4 spinal cord injury. Which assessment finding is a priority to address? A. Subjective statements of depression B. Stage 2 pressure ulcers on both heels C. Urine output for 8 hours of 480 mL, cloudy D. Blood pressure 200/100 mm Hg, throbbing headache

D. Blood pressure 200/100 mm Hg, throbbing headache

The nurse is monitoring a client who is experiencing vomiting and diarrhea for signs of dehydration. What is the best indication that the client is dehydrated? A. Sodium: 135 mEq/L (135 mmol/L) B. Hemoglobin: 10 g/dL (100 mmol/L) C. Urine specific gravity: 1.005 D. Blood urea nitrogen (BUN): 30 mg/dL (10.7 mmol/L)

D. Blood urea nitrogen (BUN): 30 mg/dL (10.7 mmol/L)

The nurse is assessing a child with increased intracranial pressure who has been exhibiting decorticate posturing. On assessment, the nurse notes extension of the upper and lower extremities with internal rotation of the upper arms, wrists, knees, and feet. How should the nurse interpret the child's condition? A. Is unchanged B. Has improved C. Indicates decreased intracranial pressure D. Indicates a deterioration in neurological function

D. Indicates a deterioration in neurological function

The nurse is interpreting the results of a tuberculin skin test (Mantoux test) on a child taking immunosuppressive doses of corticosteroids. Which interpretation indicates a positive result? A. Induration of 0 millimeters (mm) B. Induration of 1 mm C. Induration of 3 mm D. Induration of 6 mm

D. Induration of 6 mm

The client is brought from a burning building and has hard leathery black skin over the entire chest and both arms. What is the client's primary need in the emergency department? A. Analgesics B. Debridement C. Tetanus immunization D. Intravenous (IV) fluids

D. Intravenous (IV) fluids

The nurse is caring for a client who has a history of diverticulitis. The client now reports left lower quadrant pain and shoulder pain. The abdomen is board-like and tender; the client's temperature is 100.4° F (38° C). What is the client's priority need? A. An analgesic B. An antiemetic C. An antipyretic D. Intravenous fluids

D. Intravenous fluids

An antepartum client is diagnosed with bacterial vaginosis. The nurse expects to note which finding on data collection of the client? A. Costovertebral angle pain B. Proteinuria and hematuria C. Hematuria and hypertension D. Itching and vaginal discharge

D. Itching and vaginal discharge

A client taking valproic acid for the management of seizure disorder reports to the laboratory for follow-up blood tests. The nurse should check the results of which laboratory test to monitor for medication toxicity? A. Glucose B. Electrolytes C. Sedimentation rate D. Liver function studies

D. Liver function studies

A client whose magnesium level is 4 mg/dL (1.6 mmol/L) is being treated for the imbalance. The nurse determines that the electrolyte imbalance is resolving if the client has relief from which sign or symptom characteristic of this electrolyte imbalance? A. Tetany B. Twitches C. Muscular excitability D. Loss of deep tendon reflexes

D. Loss of deep tendon reflexes

When reviewing a client's laboratory results, which result should be of most concern to the nurse? A. Sodium 143 mEq/L B. Phosphate 2.6 mg/dL C. Potassium 5 mEq/L D. Magnesium 1.1 mg/dL

D. Magnesium 1.1 mg/dL

The nurse is assigned to care for a client in labor who has a diagnosis of sickle cell anemia. The nurse administers oxygen to the client and should plan for implementation of which additional measure to prevent a sickling crisis from occurring? A. Reassure the client. B. Maintain strict asepsis. C. Monitor the temperature. D. Maintain adequate hydration.

D. Maintain adequate hydration.

Following an open appendectomy procedure, a school-aged child complains of nausea. What is the nurse's priority action? A. Remove the nasogastric (NG) tube. B. Provide meticulous skin care when changing the abdominal dressing. C. Insert a second intravenous (IV) line for the administration of antiemetics. D. Maintain the child's nothing-by-mouth (NPO) status and administer prescribed intravenous (IV) fluids.

D. Maintain the child's nothing-by-mouth (NPO) status and administer prescribed intravenous (IV) fluids.

A client with chronic kidney disease returns to the nursing unit after receiving his second hemodialysis treatment; the nurse is monitoring the client closely for signs of disequilibrium syndrome. What is a sign of this syndrome? A. Irritability B. Tachycardia C. Hypothermia D. Mental confusion

D. Mental confusion

The nurse is entering a client's room wearing a gown and gloves. The nurse would don personal protective equipment for contact precautions when preparing to provide care to a client with which infection? A. Rubella B. Varicella C. Pulmonary tuberculosis (TB) D. Methicillin-resistant Staphylococcus aureus (MRSA)

D. Methicillin-resistant Staphylococcus aureus (MRSA)

A client is diagnosed with myasthenia gravis. The nurse reviews the client's physical examination report in the medical record and expects to note documentation of which clinical manifestation of this disorder? A. Pruritus B. Tachycardia C. Skin irritation D. Muscle weakness

D. Muscle weakness

The teaching plan for a child diagnosed with human immunodeficiency virus (HIV) infection should include which instruction? A. Limit all outside activities. B. Immunizations are contraindicated. C. Antiretroviral therapy can be discontinued in adulthood. D. Notify the health care provider (HCP) for chest congestion and cough.

D. Notify the health care provider (HCP) for chest congestion and cough.

You are working with a college student who is planning to become sexually active. She is requesting a reliable method of birth control that could be easily discontinued if necessary. Which option should be given the strongest recommendation? A. Intrauterine device (IUD) B. Coitus interruptus C. Natural family planning D. Oral contraceptive pills

D. Oral contraceptive pills

A hospitalized client receiving ganciclovir has acquired immunodeficiency virus (AIDS) and cytomegalovirus (CMV) retinitis. The nurse should implement which intervention during the time the client is taking this medication? A. Limit fluid intake. B. Monitor blood glucose levels for elevation. C. Ensure the medication is taken on an empty stomach only. D. Provide the client with a soft toothbrush and an electric razor.

D. Provide the client with a soft toothbrush and an electric razor.

A client with acquired immunodeficiency syndrome (AIDS) is suspected of having cutaneous Kaposi's sarcoma. The nurse prepares the client for which test that will confirm the presence of this type of sarcoma? A. Liver biopsy B. Sputum culture C. White blood cell count D. Punch biopsy of the cutaneous lesions

D. Punch biopsy of the cutaneous lesions

The nurse receives report at the beginning of the shift about a client with an intrauterine fetal demise. On assessment of the client, the nurse expects to note which finding? A. Intractable vomiting and dehydration B. Elevated blood pressure, proteinuria, and edema C. Uterine size greater than expected for gestational age D. Regression of pregnancy symptoms and absence of fetal heart tones

D. Regression of pregnancy symptoms and absence of fetal heart tones

The nurse is caring for a client with a brain injury to the brainstem. The nurse should monitor which parameter as the priority? A. Urine output B. Electrolyte results C. Peripheral vascular status D. Respiratory rate and rhythm

D. Respiratory rate and rhythm

Nursing management of the client with syndrome of inappropriate antidiuretic hormone (SIADH) includes which intervention? A. Administration of potassium supplements B. Insertion of a nasogastric tube for gastric suction C. Elevation of the head of the bed to at least 45 degrees D. Restriction of fluid intake to 1000 mL or less per day

D. Restriction of fluid intake to 1000 mL or less per day

The nurse is providing a yearly summer educational session to parents in a local community. The topic of the session is preventive and treatment measures for poison ivy. Which instruction should the nurse provide to the parents if the child comes into contact with poison ivy? A. Immediately report to the emergency department. B. Avoid becoming concerned if a rash is not noted on the skin. C. Apply calamine lotion immediately to the exposed skin areas. D. Shower the child immediately, lathering and rinsing the child several times.

D. Shower the child immediately, lathering and rinsing the child several times.

The nurse is developing a dietary plan for a client with primary hypothyroidism. The nurse should include which most appropriate food items in the plan? A. Organ meat, carrots, and skim milk B.Seafood, spinach, and cream cheese C. Peanut butter, avocado, and red meat D. Skim milk, apples, and whole-grain bread

D. Skim milk, apples, and whole-grain bread

A newborn infant with a diagnosis of subdural hematoma is admitted to the newborn nursery. The nurse should perform which action to assess for the major symptom associated with subdural hematoma? A. Monitor the urine for blood. B. Monitor the urinary output pattern. C. Test for contractures of the extremities. D. Test for equality of extremities when stimulating reflexes.

D. Test for equality of extremities when stimulating reflexes.

The nurse is assigned to care for a client with right-sided hemiparesis. The nurse plans the care of the client with what knowledge in mind? A. The client has lost the ability to swallow or speak. B. The client will still be able to bathe and ambulate independently. C. The client will exhibit complete paralysis of both upper and lower extremities. D. The client has weakness in the upper extremity and lower extremity and may exhibit involvement of the face and tongue.

D. The client has weakness in the upper extremity and lower extremity and may exhibit involvement of the face and tongue.

The nurse is monitoring a client for signs and symptoms of hypocalcemia. Which finding is an indication of this electrolyte imbalance? A. Lethargy B. Confusion C. Irritability D. Depressed sensorium

C. Irritability

Which are consequences of cold stress the nurse may observe in a newborn infant? Select all that apply. A. Jaundice B. Hypoglycemia C. Cephalhematoma D. Erythema toxicum E. Respiratory distress

A. Jaundice B. Hypoglycemia E. Respiratory distress

The nurse is caring for a client receiving infusion therapy. What are the most common reasons for using infusion therapy on clients? Select all that apply. A. Administration of medications B. Decreasing costs of health care C. Replacement of blood or blood products D. Decreasing length of stay of hospitalized clients E. Maintenance of fluid balance or correct of fluid imbalance F. Maintenance of electrolyte or acid-base balance or correction of electrolyte or acid-base imbalance

A. Administration of medications C. Replacement of blood or blood products E. Maintenance of fluid balance or correct of fluid imbalance F. Maintenance of electrolyte or acid-base balance or correction of electrolyte or acid-base imbalance

The client is a 30-year-old gravida 5 para 5 who is being seen for a postpartum visit 6 weeks after delivery. She has no medical, surgical, or gynecologic problems. She is breastfeeding and tells the nurse she wants no more children and would like to use the most effective method of birth control, but she does not want sterilization. Which contraceptive method would be most appropriate for this client? A. An intrauterine device B. An injectable hormone C. The progestin only birth control pill D. The estrogen and progestin contraceptive patch

A. An intrauterine device

Which medication classifications are examples of chemical restraints? Select all that apply. A. Antianxiety B. Antipsychotic C. Antidepressant D. Nonsedative hypnotic E. Nonsteroidal anti-inflammatory drugs (NSAIDs)

A. Antianxiety B. Antipsychotic C. Antidepressant

A clear liquid diet has been prescribed for a client. The nurse should offer which item to the client? A. Apple juice B. Orange juice C. Tomato juice D. Ice cream without nuts

A. Apple juice

The client is 7 weeks pregnant and has chosen to have genetic testing, which reveals that she is a carrier of the cystic fibrosis gene. Because cystic fibrosis is an autosomal recessive inheritance disorder, what would be the next step in the testing process? A. Arrange testing of the father of the baby. B. Arrange invasive prenatal testing of the fetus. C. Provide referral to the Cystic Fibrosis Foundation for emotional support. D. Advise the parents to prepare for the birth of an infant with cystic fibrosis.

A. Arrange testing of the father of the baby.

The client is breastfeeding her first infant who is 6 hours old, and she complains of sore nipples. Which action by the nurse is most appropriate at this time? A. Assess and assist with the newborn latch to the nipple. B. Give formula feeding to newborn for two feedings to minimize nipple trauma. C. Advise the mother to pump milk from breasts and give to infant in a bottle until soreness resolves. D. Advise the mother to limit breastfeeding to 10 minutes on each breast to minimize nipple trauma.

A. Assess and assist with the newborn latch to the nipple.

What interventions should the nurse take for a client with schizophrenia to manage comorbid disorders? Select all that apply. A. Assess the client for suicidal ideations. B. Encourage the client to avoid highly spiced foods. C. Monitor and document the client's blood pressure. D. Reinforce the client's ability to implement anxiety reduction techniques. E. Support the client's efforts to stop smoking or chewing tobacco products.

A. Assess the client for suicidal ideations. C. Monitor and document the client's blood pressure. D. Reinforce the client's ability to implement anxiety reduction techniques. E. Support the client's efforts to stop smoking or chewing tobacco products.

The client is a 33-year-old gravida 2 para 1 in active labor. She is receiving epidural anesthesia. Which nursing care practices during labor represent evidence-based care for encouraging a normal birth in a client receiving epidural anesthesia? A. Assist the client to change positions every hour. B. Encourage the client to hold her breath, and count to 10 while pushing. C. Advise the client that the epidural will be discontinued in the second stage of labor to allow effective pushing. D. Encourage the client to begin pushing when she is completely dilated (10 cm) although not feeling the urge to push.

A. Assist the client to change positions every hour.

A hospitalized client with chronic kidney disease has returned to the nursing unit after a hemodialysis treatment. The nurse should check predialysis and postdialysis documentation of which parameters to determine the effectiveness of the procedure? A. Blood pressure and weight B. Weight and blood urea nitrogen C. Potassium level and creatinine levels D. Blood urea nitrogen and creatinine levels

A. Blood pressure and weight

The client is a 19-year-old woman with a recent diagnosis of chlamydiosis. She is 10 weeks pregnant at this time. Which client statement reflects an understanding of this condition? A. Both she and her partner need treatment for chlamydiosis now. B. If chlamydiosis is present at delivery, a cesarean section would be needed. C. Treatment of chlamydiosis is contraindicated in the first trimester of pregnancy. D. Chlamydiosis is an infection that recurs throughout pregnancy despite treatment.

A. Both she and her partner need treatment for chlamydiosis now.

What are the common characteristics of bulimia nervosa? Select all that apply. A. Chronic disorder B. Episodes of binge eating C. Part of anorexia nervosa D. Begins before adolescence E. Periods of restrictive eating F. Evaluation of self by body weight and shape

A. Chronic disorder B. Episodes of binge eating E. Periods of restrictive eating F. Evaluation of self by body weight and shape

The health care provider writes a prescription for zidovudine for a client with human immunodeficiency virus (HIV) who was admitted to the hospital. The nurse should contact the health care provider and verify the prescription if which finding is noted in the assessment data? A. Cirrhosis of the liver B. Complaints of diarrhea C. History of Crohn's disease D. Complaints of mucus in the stool

A. Cirrhosis of the liver

Which of the following processes have the strongest links to intracranial regulation? (Select all that apply.) A. Cognition B. Mobility C. Oxygenation D. Perfusion E. Safety

A. Cognition B. Mobility C. Oxygenation D. Perfusion

The nurse is admitting a 6-month-old infant with respiratory syncytial virus (RSV) bronchiolitis. Which type of precautions should the nurse implement? A. Contact and droplet B. Contact and airborne C. Droplet and airborne D. Standard precautions only

A. Contact and droplet

The nurse is assessing a client who has been admitted with increased intracranial pressure (ICP). The neurological assessment of the client reveals the following data: Glasgow Coma Scale (GCS) score 10. Client opens eyes to pain, is confused with inappropriate words, and is able to localize pain. Pupils are equal with sluggish response to penlight. Respiratory assessment reveals sonorous respirations; the nurse auscultates crackles in bilateral lung bases. Oxygen saturation is 93% on room air, at 15 degrees head elevation. The nurse should implement which immediate intervention based on this assessment? A. Elevate the head of bed (HOB) to 30 degrees. B. Insert an oral airway for suctioning. C. Perform oral suctioning. D. Administer oxygen.

A. Elevate the head of bed (HOB) to 30 degrees.

The nurse is instructing a new unlicensed assistive personnel (UAP) on ways to prevent infections in clients with catheters. What tips should the nurse include in her teaching? Select all that apply. A. Encourage intake of fluids. B. The catheter will be removed as soon as clinically possible. C. Empty the drainage bag every 8 to 10 hours, if necessary. D. It is acceptable to clamp a catheter for any length of time. E. Avoid raising the drainage bag above the level of the bladder.

A. Encourage intake of fluids. B. The catheter will be removed as soon as clinically possible. E. Avoid raising the drainage bag above the level of the bladder.

The nurse managing the care of a client diagnosed with schizophrenia should include which intervention into the client's plan of care after being prescribed a conventional (non-atypical) antipsychotic medication? Select all that apply. A. Encourage the client to chew gum. B. Assess the client for possible urinary retention. C. Educate the client to the increased risk of developing hyperglycemia. D. Monitor the client's menu selections to ensure adequate fiber consumption. E. Provide the client with sunglasses when being taken outdoors for recreational walks.

A. Encourage the client to chew gum. B. Assess the client for possible urinary retention. D. Monitor the client's menu selections to ensure adequate fiber consumption. E. Provide the client with sunglasses when being taken outdoors for recreational walks.

The nurse is caring for a client with a recent head injury, and the client begins to experience a tonic-clonic seizure. Which nursing intervention is a priority for this client? A. Ensure a patent airway. B. Reassure and orient the client. C. Protect the client's head by holding it upright to avoid injury. D. Obtain a complete set of vital signs, including oxygen saturation.

A. Ensure a patent airway.

The nurse is performing an assessment on a client with a diagnosis of Bell's palsy. The nurse should expect to observe which finding in the client? A. Facial drooping B. Periorbital edema C. Twitching on the affected side of the face D. Ptosis of the eyelid and closure of the eye

A. Facial drooping

Violence can occur anywhere in the hospital but is most likely to occur in what areas? Select all that apply. A. Geriatric units B. Pediatric units C. Psychiatric units D. Security departments E. Emergency departments F. Medical-surgical waiting rooms

A. Geriatric units C. Psychiatric units E. Emergency departments

The nurse is developing a plan of care for a client who is experiencing homonymous hemianopsia following a stroke (brain attack). The nurse documents interventions that will promote a safe environment knowing that in this disorder the client experiences which symptom? A. Has a visual loss in the same half of the visual field of each eye B. Has lost the ability to recognize familiar objects through the senses C. Is unable to carry out a skilled act, such as dressing, in the absence of paralysis D. Has paralysis of the sympathetic nerves of the eye causing ocular manifestations

A. Has a visual loss in the same half of the visual field of each eye

The nurse working in a prenatal clinic is reviewing the records of clients scheduled for prenatal visits. The nurse interprets that the client most at risk for abruptio placentae is the one who presents which clinical picture? A. Has maternal hypertension B. Is 26 years old and is a primipara C. Takes folic acid supplements daily D. Rides an exercise bike for 30 minutes three times weekly

A. Has maternal hypertension

What are the guidelines that commonly define psychiatric home care? Select all that apply. A. Homebound status of the client B. Presence of a psychiatric/mental health diagnosis C. Need for the skills of a psychiatric registered nurse (RN) D. Development of a plan of care under prescriptions of a health care provider (HCP) E. Providing for the technical skills of a home-care nurse as it pertains to the physiological needs of the client

A. Homebound status of the client B. Presence of a psychiatric/mental health diagnosis C. Need for the skills of a psychiatric registered nurse (RN) D. Development of a plan of care under prescriptions of a health care provider (HCP)

The nurse is reviewing the laboratory results of a client with cancer and notes that the calcium level is 14 mg/dL (3.5 mmol/L). The nurse determines that this calcium level is consistent with which oncological emergency? A. Hypercalcemia B. Tumor lysis syndrome C. Spinal cord compression D. Superior vena cava syndrome

A. Hypercalcemia

The nurse reviews the serum laboratory study results for a client taking hydrochlorothiazide. The nurse should monitor for which most frequent medication side effect? A. Hypokalemia B. Hypocalcemia C. Hypernatremia D. Hyperphosphatemia

A. Hypokalemia

20-year-old woman comes for preconceptual counseling. She wants to get pregnant soon. Which of the following health-promoting habits would have the highest priority at this time? A. Immediate tobacco cessation B. Getting daily exercise C. Stopping all caffeine D. Avoidance of sweets

A. Immediate tobacco cessation

A young adult is being evaluated for a possible eating disorder. Which nursing intervention is most directly related to a commonly observed complication related to bulimia nervosa? A. Increasing potassium-rich foods in the daily diet B. Adding fiber to the diet to help minimize constipation C. Medicating the client for migraine headaches as they occur D. Monitoring the client for signs of developing contact dermatitis

A. Increasing potassium-rich foods in the daily diet

The nurse reviews the assessment data on a client with a head injury and notes that the client's intracranial pressure reading is 10 mm Hg. Based on this finding, the nurse determines that the client's intracranial pressure reading indicates which finding? A. Is normal B. Is elevated C. Requires health care provider notification D. Needs to be reduced with aggressive treatment measures

A. Is normal

A client with a cerebral aneurysm is being admitted to the neurological unit, and the nurse prepares to place the client on aneurysm precautions. Which interventions relate to this type of precautions? Select all that apply. A. Keep the room slightly darkened. B. Limit visitors' time to short periods. C. Place the client in a supine position. D. Place the client in a quiet semiprivate room. E. Provide physical and hygienic care for the client. F. Restrict radio, television, and reading materials for the client.

A. Keep the room slightly darkened. B. Limit visitors' time to short periods. E. Provide physical and hygienic care for the client. F. Restrict radio, television, and reading materials for the client.

The nurse develops a plan of care for a client with a new diagnosis of Graves' disease. The nurse should include which intervention in the plan of care? A. Keep the room temperature cool. B. Place extra blankets on the client's bed. C. Provide a diet low in calories and protein. D. Encourage frequent ambulation and other physical activities.

A. Keep the room temperature cool.

The client is in active labor at 41 weeks' pregnancy. Her membranes have just ruptured spontaneously. The nurse notes the presence of thick meconium. Which actions by the nurse are most appropriate at this time? Select all that apply. A. Listen to the fetal heart tones. B. Prepare for immediate delivery. C. Inform the neonatal team of the presence of meconium near the time of delivery. D. Prepare wall suction, laryngoscope blade, and endotracheal tubes for possible use at delivery. E. Inform the parents that the infant will need to go to the neonatal intensive care unit (NICU) after birth because of meconium aspiration.

A. Listen to the fetal heart tones. C. Inform the neonatal team of the presence of meconium near the time of delivery. D. Prepare wall suction, laryngoscope blade, and endotracheal tubes for possible use at delivery.

The nurse is performing an assessment on a client with a diagnosis of left-sided heart failure. Which assessment would elicit specific information regarding the client's left-sided heart function? A. Listening to lung sounds B. Monitoring for organomegaly C. Assessing for jugular vein distention D. Assessing for peripheral and sacral edema

A. Listening to lung sounds

The nurse is caring for a patient experiencing an immune response. She assesses the patient for development of a hyperimmune response while knowing that cytotoxic T cells are responsible for which action? A. May kill healthy cells along with foreign antigens. B. Are the most prevalent type of T lymphocyte. C. Can suppress the immune response. D. Diminish dendritic cell function.

A. May kill healthy cells along with foreign antigens.

The nurse is reviewing medication prescriptions for a newly admitted client with type 1 diabetes mellitus. Which prescription should the nurse clarify with the health care provider? A. Metformin 500 mg by mouth two times per day B. Insulin glargine 10 units subcutaneously at bedtime C. Lispro insulin sliding scale before meals and at bedtime D. Dextrose 50% ampule intravenous push for blood glucose less than 50 mg/dL

A. Metformin 500 mg by mouth two times per day

The nurse is admitting a client with suspected bacterial meningitis. Which interventions are the most appropriate for the nurse to implement? Select all that apply. A. Monitor for seizure activity. B. Implement seizure precautions. C. Monitor temperature once per shift. D. Instruct the family to let the client rest and abstain from visiting. E. Position the client with the head slightly extended to provide pain relief.

A. Monitor for seizure activity. B. Implement seizure precautions. E. Position the client with the head slightly extended to provide pain relief.

The nurse is reviewing the laboratory results of a client and notes that the client has a magnesium level of 1.3 mg/dL (0.5 mmol/L). What is the most appropriate nursing action? A. Monitor the client for dysrhythmias. B. Encourage increased intake of phosphorus antacids. C. Instruct the client to increase the consumption of foods low in magnesium. D. Consult with the health care provider about the need to discontinue magnesium intake.

A. Monitor the client for dysrhythmias.

The nurse is assessing a person for mania. What general characteristics are considered when assessing for mania? Select all that apply. A. Mood B. Behavior C. Personality D. Cognitive function E. Thought processes and speech patterns

A. Mood B. Behavior D. Cognitive function E. Thought processes and speech patterns

What is the most important sign or symptom to report to the health care provider for a client who has chronic obstructive pulmonary disease (COPD)? A. Mucus has become yellow-green. B. The client's hemoglobin is 14 g/dL (140 mmol/L). C. The client's body mass index (BMI) is 18. D. The client's inspiration:expiration ratio is 1:3.

A. Mucus has become yellow-green.

The client with cirrhosis has had hepatitis C for 20 years. What assessment finding is most important to investigate? A. New onset confusion B. Jaundice with pruritus C. Elevated liver enzyme levels D. Bilirubin that is three times the normal value

A. New onset confusion

The nurse is assigned to care for a client who is taking furosemide. The nurse determines it is important to monitor the client's intake and output (I&O). When measuring output, which actions should the nurse take? Select all that apply. A. Note all types of output on the I&O sheet. B. Perform hand hygiene, and apply sterile gloves. C. Ask for the client's permission to measure output. D. Use a graduated cylinder to obtain measurement. E. Write down the amount immediately after measuring.

A. Note all types of output on the I&O sheet. D. Use a graduated cylinder to obtain measurement. E. Write down the amount immediately after measuring.

The client is prescribed to receive Humulin N insulin at 0700. To reduce the risk of hypoglycemia, the nurse should perform which action? A. Offer the client a snack around 10:00. B. Hold the insulin dose if the client's blood glucose is less than 150 mg/dL (8.3 mmol/L). C. Question the health care provider regarding the time of the dosing schedule. D. Keep 50% dextrose intravenous (IV) solution available at the bedside at all times.

A. Offer the client a snack around 10:00.

The nurse is caring for a client with external otitis. Which prescriptions should the nurse anticipate implementing as treatment for this condition? Select all that apply. A. Oral analgesics B. Application of heat C. Systemic antibiotics D. Application of ice pack E. Topical steroid eardrops

A. Oral analgesics B. Application of heat E. Topical steroid eardrops

A female who has recently immigrated to the United States from Sudan is being treated for depression related to her inability to become pregnant. The client volunteers that she has a history of urinary tract infections. Which statement by the nurse demonstrates an understanding of how culture can affect both physical and mental health? A. "When did these urinary tract infections first begin?" B. "Does your culture support surgically altering female genitals?" C. "Is depression common among the other women in your community?" D. "Have you ever been told the infections could be related to your infertility issues?"

B. "Does your culture support surgically altering female genitals?"

A middle-aged man reports that although he is interested in having a sexual relationship with his wife, he has difficulty sustaining an erection. The nurse starts the assessment interview by asking which initial question? A. "Would you say you have a good marital relationship?" B. "Have you ever been told you have high blood pressure?" C. "Can you explain what you mean by 'sustaining an erection'?" D. "Does this problem occur when you are under an unusual amount of stress?"

B. "Have you ever been told you have high blood pressure?"

A client with hypertension and diabetes mellitus is to start taking propranolol as part of his daily medication regimen. Which statement indicates an understanding of this medication? A. "This medication will result in needing less insulin each day." B. "I will be sure to pay close attention for symptoms of hypoglycemia." C. "This medication will decrease my blood glucose, so I will need to check it more often." D. "I should notice a decrease in my blood glucose within 1 month of starting this medication."

B. "I will be sure to pay close attention for symptoms of hypoglycemia."

The nurse provided discharge instructions to a client who has had an external fixator placed on the right wrist. Which statement, if made by the client, indicates a need for further teaching? A. "I can still exercise my arm with the external fixator on it." B. "Yellow/green drainage is expected in the first 48 to 72 hours after surgery." C. "I will clean the pin sites according to the approved directions given to me." D. "I will need to monitor the pin sites every 8 to 12 hours for drainage, color, odor, and severe redness."

B. "Yellow/green drainage is expected in the first 48 to 72 hours after surgery."

Which surgical client is at increased risk for a wide temperature variation during surgery? A. A 19-year-old client scheduled for arthroscopy B. A 3-month-old infant scheduled for hernia repair C. A 62-year-old client scheduled for hip replacement D. A 54-year-old female scheduled for total abdominal hysterectomy

B. A 3-month-old infant scheduled for hernia repair

A patient presents to the clinic with observable edema and erythema of the left forearm. A brief history reveals no exposure to potential irritating agents. On palpation, the nurse finds the area very warm and tender. What is the most likely cause of the patient's symptoms? A. An allergic reaction B. A complement cascade C. IgE reactions D. Clonal diversity

B. A complement cascade

What factor affecting heat loss by convection should the nurse be aware of when giving a client a bath? A. The time the bath was taken B. A draft in the client's bathroom C. A cool temperature in the bathroom D. Perspiration due to the water temperature

B. A draft in the client's bathroom

The nurse is assessing a group of clients for the risk factors for acquiring pneumonia during hospitalization. The nurse determines that which client is least likely at risk? A. An older client with diabetes mellitus B. A postoperative client who is ambulating C. A client with a spinal cord injury who is immobile D. A client with human immunodeficiency virus (HIV)

B. A postoperative client who is ambulating

The client has a T3 spinal cord injury and is brought to the emergency department. The nursing assessment reveals blood pressure 70/40 mm Hg, pulse 50 beats/min, and respirations 18 breaths/min, and the nurse suspects neurogenic shock. The client's skin is warm, dry, and pink. What action should the nurse prepare to take first? A. Assess cranial nerves X and XI. B. Administer intravenous isotonic fluids. C. Place the client in Trendelenburg's position. D. Perform a Glasgow Coma Scale assessment.

B. Administer intravenous isotonic fluids.

The nurse is planning care for a child with Kawasaki disease. Which interventions should the nurse plan to implement? Select all that apply. A. Restricting fluids B. Administering aspirin as prescribed C. Avoiding the use of bright overhead lights D. Administering penicillin G benzathine as prescribed E. Administering intravenous immune globulin (IVIG) as prescribed

B. Administering aspirin as prescribed C. Avoiding the use of bright overhead lights E. Administering intravenous immune globulin (IVIG) as prescribed

A child develops a severe anaphylactic reaction to penicillin. Which prescribed action should the nurse prepare to implement first? A. Initiation of intravenous fluid B. Administration of epinephrine C. Administration of a vasopressor D. Administration of diphenhydramine

B. Administration of epinephrine

The nurse is planning care for a child with rheumatic fever (RF). Which interventions should the nurse plan to implement? Select all that apply. A. Strict bed rest B. Administration of salicylates as prescribed C. Maintenance of contact isolation precautions D. Home care teaching about taking prophylactic antibiotics E. Administration of penicillin G benzathine as prescribed

B. Administration of salicylates as prescribed D. Home care teaching about taking prophylactic antibiotics E. Administration of penicillin G benzathine as prescribed

The nurse is caring for a client who has been taking antipsychotic medications and is exhibiting extrapyramidal symptoms (EPS). What are EPS symptoms? Select all that apply. A. Paranoia B. Akathisia C. Parkinsonism D. Acute dystonia E. Severe anxiety F. Tardive dyskinesia (TD)

B. Akathisia C. Parkinsonism D. Acute dystonia F. Tardive dyskinesia (TD)

The client is a 38-year-old woman at 15 weeks' pregnancy who is considering genetic amniocentesis. Which statement by the nurse represents accurate information regarding this procedure? A. Amniocentesis leads to a high risk of spontaneous abortion. B. Amniocentesis can screen for some specific genetic conditions. C. Amniocentesis should be done in all women over the age of 35 years. D. Amniocentesis is a safe and accurate way to find out if the baby is healthy.

B. Amniocentesis can screen for some specific genetic conditions.

An adult client with heart failure (HF) has been prescribed furosemide for fluid overload and is at risk for potassium imbalance. What actions should the nurse teach the client to prevent this imbalance? Select all that apply. A. Drink at least 3.5 L of fluid daily. B. Take the potassium supplements that have been prescribed. C. Decrease the dose of the medication by half if symptoms appear. D. If symptoms of potassium imbalance appear, take the prescribed digoxin every other day. E. Know the symptoms of decreased potassium levels, for example, muscle weakness and heart irregularity. F. Eat foods high in potassium, for example, tomatoes, beans, prunes, avocados, bananas, strawberries, and lettuce.

B. Take the potassium supplements that have been prescribed. E. Know the symptoms of decreased potassium levels, for example, muscle weakness and heart irregularity. F. Eat foods high in potassium, for example, tomatoes, beans, prunes, avocados, bananas, strawberries, and lettuce.

The nurse is reviewing the assessment findings and laboratory results of a child diagnosed with new-onset glomerulonephritis. Which finding should the nurse most likely expect to note? A. Hypotension B. Tea-colored urine C. Low serum potassium D. Elevated creatinine levels

B. Tea-colored urine

The client had a cesarean section 12 hours ago following a prolonged labor. Spinal anesthesia was used. Which finding indicates the need for immediate further assessment? A. The client's temperature is 38° C. B. The client cannot move her legs freely. C. The client requires assistance to breastfeed infant. D. Urinary catheter output is 40 mL/hr.

B. The client cannot move her legs freely.

The nurse is listing goals for a client with a thoracic 4 (T4) vertebral spinal cord injury to prevent autonomic dysreflexia. Which goal is most appropriate to prevent this life-threatening complication? A. The client wears elastic support stockings at all times. B. The client performs self-catheterization every 6 hours. C. The client turns, coughs, and deep breathes every 2 hours. D. The client takes medication to relieve muscle spasms daily.

B. The client performs self-catheterization every 6 hours.

The client presents to the emergency department with severe vomiting and diarrhea and is diagnosed with gastroenteritis. Which occurrence is most important before allowing the client to be discharged home? A. Bowel sounds within normal limits B. The client's ability to tolerate oral fluids C. Administration of antidiarrheal medication D. The client's ability to demonstrate knowledge of prescribed diet

B. The client's ability to tolerate oral fluids

The nurse is caring for a client who has a fever and is diaphoretic. The nurse monitors the client's intake and output and expects which finding? A. The client's urine is diluted. B. The client's output is decreased. C. The client's urine production is increased. D. The majority of the client's fluid is excreted through the skin.

B. The client's output is decreased.

A client is hospitalized and is being treated for active tuberculosis (TB). The nurse should provide which information to the client related to preventing transmission to family members? A. The family members will need to take medication to prevent infection even though the client will not be contagious after 1 continuous week of medication therapy. B. The family members will need to take medication to prevent infection even though the client will not be contagious after 2 to 3 consecutive weeks of medication therapy. C. The family members will not need to take medication to prevent infection because the tuberculosis cannot be transmitted once the client has taken tuberculosis medications for 1 month. D. The family members will not need to take medication to prevent infection because the tuberculosis cannot be transmitted once the client has taken tuberculosis medications for 6 consecutive weeks.

B. The family members will need to take medication to prevent infection even though the client will not be contagious after 2 to 3 consecutive weeks of medication therapy.

The nurse is reviewing the fetal monitor record of a client who is in active labor at 37 weeks' pregnancy. Which finding would require the nurse to notify the health care provider immediately? A. The contractions are every 2½ minutes. B. The fetal heart rate has been 175 beats/min for 20 minutes. C. The fetal heart rate has been 115 beats/min for 30 minutes. D. The fetal heart rate decelerates 15 beats/min with each contraction but recovers to baseline by the end of the contraction.

B. The fetal heart rate has been 175 beats/min for 20 minutes.

A client is being treated for behaviors associated with extreme exhibitionism. In addressing the client's need for medication education, the nurse plans to discuss what medication? A. The vasodilator sildenafil citrate B. The serotonergic antidepressant fluoxetine C. The general classification of benzodiazepines D. The general classification of tricyclic antidepressant

B. The serotonergic antidepressant fluoxetine

The nurse is assembling the supplies for a paracentesis. Which action would require the nurse to discard the equipment and prepare a new field of supplies? A. Sterile gloves are donned to open the supplies. B. The sterile tray is left open unattended for 5 minutes. C. Sterile gloves are placed on the sterile tray of equipment. D. A disposable 18-gauge, 1.5-inch needle is placed on the sterile tray.

B. The sterile tray is left open unattended for 5 minutes.

Intracranial regulation would be a priority concern for the nurse caring for a patient with which admitting diagnosis? A. Failure to thrive B. Traumatic brain injury C. Upper respiratory infection D. Urinary tract infection

B. Traumatic brain injury

Although sexual activity is considered a normative process, some individuals place themselves at increased risk for negative consequences related to this process. Which nonsexual behavior is likely to increase risk-taking activities? A. Having multiple sexual partners B. Using alcohol, marijuana, or illicit substances C. Having gay, lesbian, or bisexual partners D. Refraining from safe-sex practices such as condom use

B. Using alcohol, marijuana, or illicit substances

A postpartum mother complains of severe pain and an intense feeling of swelling and pressure in the vaginal area. The nurse should check which anatomical area immediately? A. Vagina for lacerations B. Vulva for a hematoma C. Rectum for hemorrhoids D. Episiotomy site for drainage

B. Vulva for a hematoma

A client has a wound infection that has required the client to be placed on contact precautions. What other intervention should the nurse prepare to implement? A. Preventing breaks in the chain of infection B. Identifying organisms growing in the cultures C. Collecting specimens from the client's wound D. Providing the client with a diet low in carbohydrates

C. Collecting specimens from the client's wound

The nurse is reviewing the erythrocyte sedimentation rate (ESR) of a patient to determine which significant finding? A. Determines specific causes of inflammation B. Identifies the location of inflammation within the body C. Confirms the nonspecific presence of inflammation D. Indicates a diagnosis of systemic lupus

C. Confirms the nonspecific presence of inflammation

Which of the clients who have come into the obstetrics triage area of the hospital should the nurse assess first? A. A 17-year-old gravida 1 at 39 weeks' pregnancy with contractions every 7 minutes who is crying loudly B. A 32-year-old gravida 2 para 1 at 38 weeks' pregnancy with clear fluid leaking from the vagina but no contractions C. A 42-year-old gravida 1 at 39 weeks' pregnancy complaining of a headache, visual changes, and epigastric pain D. A 26-year-old gravida 2 para 1 at 27 weeks' pregnancy noting vaginal spotting following sexual intercourse today

C. A 42-year-old gravida 1 at 39 weeks' pregnancy complaining of a headache, visual changes, and epigastric pain

The nurse educator is teaching a group of nursing students about conditions that require a child to receive prophylactic antibiotic therapy before a dental procedure. Which child should the nurse include in the category of needing prophylactic antibiotic therapy before a dental procedure? A. A child with a previous history of rheumatic fever B. A child with a previous history of Kawasaki disease C. A child with a previous episode of infective endocarditis D. A child with a history of a patent ductus arteriosus repair at birth

C. A child with a previous episode of infective endocarditis

Which child must be placed in a private room when admitted to the hospital? A. A client with conjunctivitis B. A client diagnosed with scabies C. A client diagnosed with chickenpox (varicella) D. A client admitted with methicillin-resistant Staphylococcus aureus (MRSA)

C. A client diagnosed with chickenpox (varicella)

The nurse is screening clients for their risk of developing type 2 diabetes mellitus. The nurse should consider which client at greatest risk? A. A client with hyperthyroidism complaining of excessive sweating B. A client with hyperlipidemia whose mother has type 2 diabetes mellitus C. A client who is obese with complaints of urinary frequency and hunger D. A client with a body mass index (BMI) of 27 with a history of hypertension and anxiety

C. A client who is obese with complaints of urinary frequency and hunger

The nurse is developing a teaching plan for the client with viral hepatitis. The Nurse should list which item in the plan? A. Consume three large meals daily B. The diet should be low in calories C. Activity should be limited to prevent fatigue D. Alcohol intake should be limited to 2 oz (60 mL) per day

C. Activity should be limited to prevent fatigue

The nurse is reviewing the laboratory results of a client and notes that the serum potassium level is 5.5 mEq/L (5.5 mmol/L). The nurse understands that this laboratory result would be noted in a client who had which condition? A. Diarrhea B. Diabetes insipidus C. Addison's disease D. Dumping syndrome

C. Addison's disease

The client is a 39-year-old gravida 1, who has a busy life as a consultant, which requires frequent travel. She is at 28 weeks' pregnancy and has begun to have vaginal bleeding. She has just been told she has placenta previa and that she will need to be on bed rest in the hospital for the present time until all active bleeding ceases, and she will then be evaluated for possible home care. The client is crying and stating that she cannot follow this regimen of care because she has many responsibilities to fulfill at work this week, including travel to another city. She states the bleeding has decreased and she wants to go home. What action by the nurse can best assist the client to an understanding and acceptance of her diagnosis at this time? A. Tell the client that her fetus is at great risk now and that she needs to think only of her baby's well-being. B. Tell the woman that proceeding with her travel and work plans will increase the risks to both her and the fetus. C. Allow the woman to cry and listen to her express her feelings before reviewing the diagnosis with her. D. Instruct the woman in signs and symptoms of hemorrhage so that she could seek emergency department care if it were to occur while traveling.

C. Allow the woman to cry and listen to her express her feelings before reviewing the diagnosis with her.

The nurse is caring for a military veteran with post-traumatic stress disorder (PTSD) who is experiencing symptoms of avoidance of trauma-reminding phenomena. What are some manifestations of this group of phenomena? Select all that apply. A. Feelings of reality B. Neutral emotional state C. Amnesia for the event in question D. Avoidance of trauma-related activities E. Feelings of detachment or estrangement F. Persistent and exaggerated negative beliefs

C. Amnesia for the event in question D. Avoidance of trauma-related activities E. Feelings of detachment or estrangement F. Persistent and exaggerated negative beliefs

The nurse is preparing to administer medications to a client who has been admitted with a stroke (brain attack) with a residual complication of impaired swallowing. What interventions should the nurse initiate to prevent the client from aspirating? Select all that apply. A. Crush all medications, and put them in applesauce for the client to swallow. B. Let the client use straws to ease the process of swallowing and to control liquids. C. Assess the client's ability to swallow and cough, and check for presence of gag reflex. D. If unilateral weakness is present, place the medication into the stronger side of the mouth. E.Administer the pills one at a time, ensuring that each medication is properly swallowed before the next one is introduced.

C. Assess the client's ability to swallow and cough, and check for presence of gag reflex. D. If unilateral weakness is present, place the medication into the stronger side of the mouth. E.Administer the pills one at a time, ensuring that each medication is properly swallowed before the next one is introduced.

Nitrofurantoin is prescribed for a client with a urinary tract infection, and the nurse provides instructions to the client about the medication. The nurse should tell the client to contact the health care provider if which sign or symptom occurs? A. Nausea B. Diarrhea C. Chest pain D. Loss of appetite

C. Chest pain

During the administration of a blood transfusion to a client, the nurse notes the presence of crackles in the client's lung bases. On further data collection, the nurse notes that the client has distended neck veins and an increase in blood pressure. The nurse suspects that the client is experiencing what complication of the blood transfusion? A. Sepsis B. Allergic reaction C. Circulatory overload D. Transfusion reaction

C. Circulatory overload

The nurse is providing care to a client with increased intracranial pressure (ICP). Which approach is least likely to be beneficial in controlling the client's increased ICP? A. Reducing environmental noise B. Maintaining a quiet environment C. Clustering nursing activities to be done all at one time D. Maintaining the client's head elevated in a midline neutral position

C. Clustering nursing activities to be done all at one time

A female patient comes to the clinic at 8 weeks' gestation. She lives in a house beneath electrical power lines, which is located near an oil field. She drinks two caffeinated beverages a day, is a daily beer drinker, and has not stopped eating sweets. She takes a multivitamin and exercises daily. She denies drug use. Which finding in the history has the greatest implication for this patient's plan of care? A. Electrical power lines are a potential hazard to the woman and her fetus. B. Living near an oil field may mean the water supply is polluted. C. Drinking alcohol should be avoided during pregnancy because of its teratogenic effects. D. Eating sweets may cause gestational diabetes or miscarriage.

C. Drinking alcohol should be avoided during pregnancy because of its teratogenic effects.

A client with a diagnosis of suspected food poisoning is admitted to the hospital because of dehydration. The nurse should expect to note which finding on assessment of this client? A. Decreased pulse B. Increased urine output C. Dry mucous membranes D. Decreased respiratory rate

C. Dry mucous membranes

Despite the importance of sexual health to overall well-being, many nurses and patients are uncomfortable discussing issues related to sexuality. It is for this reason that the nurse must include questions regarding a sexual health history as part of a comprehensive health assessment. A 15-year-old female patient has come to the office for her annual physical and first pelvic examination. In this situation, which nursing action is most important? A. Encourage the patient to ask questions about sexuality. B. Screen for possible abuse. C. Excuse the parent. D. Ensure the patient that all information will be kept confidential.

C. Excuse the parent.

A client is brought into the emergency department with a diagnosis of early chickenpox (varicella). Of all of the equipment needed, which one is most important for the nurse to have ready at the bedside? A. Gown B. Gloves C. Facemasks D.Hand sanitizer

C. Facemasks

An infant born with hydrocephalus is to be discharged after insertion of a ventriculoperitoneal shunt. Which common complication of this type of surgery should the nurse explain to the home caregivers to prepare them for the patient's discharge? A. Excessive fluid accumulation in the abdomen B. Eyes with sclera visible above the irises C. Fever accompanied by decreased responsiveness D. Violent involuntary muscle contractions

C. Fever accompanied by decreased responsiveness

The health care provider's office nurse is assessing a client who has recently had a renal transplantation. The nurse should monitor for which signs of acute graft rejection? A. Hypotension, graft tenderness, and anemia B. Hypertension, oliguria, thirst, and hypothermia C. Fever, hypertension, graft tenderness, and malaise D. Fever, vomiting, hypotension, and copious amounts of dilute urine

C. Fever, hypertension, graft tenderness, and malaise

The client has been diagnosed with polycystic kidney disease. The nurse should assess the client for which manifestation that is most common for this disorder? A. Headache B. Hypotension C. Flank pain and hematuria D. Complaints of low pelvic pain

C. Flank pain and hematuria

In order to fully understand the concept of sexuality, it is necessary to become familiar with the terms used when discussing this topic. Which term best describes how one views oneself as masculine or feminine? A. Sexual identity B. Sexual orientation C. Gender identity D. Sexual behavior

C. Gender identity

Hypoglycemia is most likely a potential effect of which oral hypoglycemic agent? A. Acarbose B. Sitagliptin C. Glipizide D. Glucophage

C. Glipizide

The nurse is caring for a client diagnosed with delusions. What are common delusions? Select all that apply. A. Control B. Jealousy C. Grandiose D. Persecutory E. Religious or hypochondriacal ideas

C. Grandiose D. Persecutory E. Religious or hypochondriacal ideas

A client has been admitted to the hospital with a fractured pelvis sustained in a motor vehicle crash. The nurse monitors for complications and should assess the client closely for which finding in the early posttrauma period? A. Pain B. Fever C. Hematuria D. Bradycardia

C. Hematuria

What are the prominent suicidal ideation themes experienced by clients with suicidal ideation? Select all that apply. A. Anxiety B. Nervousness C. Hopelessness D. Meaninglessness E. Being out of control

C. Hopelessness D. Meaninglessness E. Being out of control

The nurse is caring for a client who is receiving a potassium-retaining diuretic. The nurse should monitor for which side effect of the medication? A. Dry skin B. Constipation C. Hyperkalemia D. Hypernatremia

C. Hyperkalemia

A CD4+ T-cell count is performed on a client who is human immunodeficiency virus (HIV) positive. The results of the test indicate a CD4+ count of 1000 cells/mm3. The nurse should interpret this test result to indicate which finding? A. That infection is likely to develop B. That the count is dangerously low C. Improvement in the client's condition D. The need for aggressive therapy with intravenous antibiotics

C. Improvement in the client's condition

The nurse is caring for a hospitalized client with acquired immunodeficiency syndrome (AIDS) who is receiving didanosine. The nurse should contact the health care provider if which laboratory result is noted that may be an indication of pancreatitis? A. Increased creatinine B. Increased potassium C. Increased serum triglycerides D. Increased serum triglycerides

C. Increased serum triglycerides

The health care provider prescribes aripiprazole for a client with a diagnosis of schizophrenia. Which nursing intervention would be therapeutic? A. Administer the medication only after meals. B. Inform the client to limit his alcohol intake to one drink each day. C. Inform the client that the medication may cause sedation and should be taken at bedtime. D. Instruct the client to increase his usual exercise pattern threefold to help with medication absorption.

C. Inform the client that the medication may cause sedation and should be taken at bedtime.

The client presents with a cough productive of green phlegm. Vital signs are 100.4° F (38° C), pulse 110 beats/min, respirations 24 breaths/min, and blood pressure 90/40 mm Hg. What is the client's priority need? A. Guaifenesin B. Acetaminophen C. Intravenous (IV) fluids D. High-Fowler's position

C. Intravenous (IV) fluids

A client in labor has been repositioned from side to side every 30 minutes. The client tells the nurse that she is tired of having to lie on her side and would like to lie on her back for a while. The nurse should take which most appropriate action? A. Tell the client that the supine position is contraindicated during labor. B. Tell the client that bed rest, lying on one side or the other, is necessary. C. Position the client supine and place a pillow under one hip to act as a wedge. D. Tell the client that the obstetrician will need to be called to obtain a prescription for lying in the supine position.

C. Position the client supine and place a pillow under one hip to act as a wedge.

An older client is recovering after a total hip replacement. Two days after surgery, the nurse is reviewing the client's laboratory values. Which value should alert the nurse to contact the client's health care provider? A. Calcium 9.0 mg/dL (2.25 mmol/L) B. Sodium 141 mEq/L (141 mmol/L) C. Potassium 3.2 mEq/L (3.2 mmol/L) D. Total protein 65 g/L (6.5 g/dL)

C. Potassium 3.2 mEq/L (3.2 mmol/L)

A client with acute kidney injury has been treated with sodium polystyrene sulfonate by mouth. The nurse should evaluate this therapy as most effective if which value was noted on follow-up laboratory testing? A. Calcium 9.8 mg/dL (2.5 mmol/L)) B. Sodium 142 mEq/L (142 mmol/L) C. Potassium 4.9 mEq/L (4.9 mmol/L) D. Phosphorus 3.9 mg/dL (1.26 mmol/L)

C. Potassium 4.9 mEq/L (4.9 mmol/L)

The nurse caring for a client following a bowel resection notes that the client is restless. The nurse takes the client's vital signs and notes that the client's pulse rate has increased and that the blood pressure has dropped significantly since the previous readings. The nurse suspects that the client is going into shock and should take which immediate action? A. Check the client's oxygen saturation level. B. Recheck the vital signs to verify the findings. C. Raise the client's legs above the level of the heart. D. Slow the rate of the intravenous (IV) fluid infusing.

C. Raise the client's legs above the level of the heart.

The client had a stroke (brain attack) and has residual Broca's aphasia. What action should the nurse include in the plan of care? A. Institute fall precautions. B. Add thickener to liquids. C. Realize the client may have difficulty speaking. D. Provide written instructions to aid client understanding.

C. Realize the client may have difficulty speaking.

A client experiencing delusions of being poisoned is admitted to the hospital. The client shows no evidence of dehydration and malnutrition at this time. The nurse prepares a plan of care for the client and should include which client need as the priority? A. Self-esteem needs B. Physiological needs C. Safety and security needs D. Love and belonging needs

C. Safety and security needs

The new nurse is reviewing the safety guidelines for nursing skills. After reviewing the skills, which guideline, identified by the nurse, requires clarification? A. Use clean gloves when you anticipate contact with body fluids. B. Use gown, mask, and eye protection when there is a risk of a splash. C. Shared equipment between clients does not need to be cleaned if clients are not contagious. D. Keep bedside table surfaces clutter free, clean, and dry when performing aseptic procedures.

C. Shared equipment between clients does not need to be cleaned if clients are not contagious.

The client with recurrent urinary tract infection (UTI) asks the nurse what measures can be taken to reduce recurrent infections. What should the nurse include in the teaching plan? Select all that apply. A. Drink orange juice daily B. Wear synthetic underwear. C. Shower instead of taking a bath. D. Drink 8 to 10 glasses of water a day. E. Hold the urine in the bladder for at least 4 to 6 hours before voiding.

C. Shower instead of taking a bath. D. Drink 8 to 10 glasses of water a day.

The nurse receives a telephone call from the hospital admission office and is told that a client with human immunodeficiency virus (HIV) will be admitted to the nursing unit. In planning infection control measures for the client, the nurse prepares to institute which type of isolation precaution? A. Droplet precautions B. Contact precautions C. Standard precautions D. Airborne precautions

C. Standard precautions

The nurse notes that the health care provider has prescribed sulfasalazine for a client. The nurse checks the nursing history form in the client's medical record for documentation of an allergy to which item? A. Shellfish B. Strawberries C. Sulfonamides D. Acetaminophen

C. Sulfonamides

The nurse is caring for a child diagnosed with bacterial conjunctivitis or "pink eye." Which are clinical manifestations specific to bacterial conjunctivitis? Select all that apply. A. Pain B. Itching C. Swollen lids D. Purulent drainage E. Inflamed conjunctiva F. Crusting of eyelids, especially on awakening

C. Swollen lids D. Purulent drainage E. Inflamed conjunctiva F. Crusting of eyelids, especially on awakening

The nurse is reviewing the diagnostic tests prescribed for a client. The nurse notes that a lupus cell preparation (LE cell prep) has been prescribed. The nurse determines that this test is used to screen primarily for which disorder? A. Histoplasmosis B. Progressive systemic sclerosis C. Systemic lupus erythematosus (SLE) D. Human immunodeficiency virus (HIV)

C. Systemic lupus erythematosus (SLE)

The nurse is admitting a client to the hospital who has experienced a stroke (brain attack), which has caused damage to the frontal lobe. Which assessment abnormality can the nurse expect to note when assessing this client? A. The client is hypothermic. B. The client exhibits receptive aphasia. C. The client exhibits expressive aphasia. D. The client has lost the ability to recognize smell.

C. The client exhibits expressive aphasia.

A client's chart makes note that she has been demonstrating "all or nothing thinking." Which client behavior supports this observation? A. The client insists she was responsible for the party being poorly attended. B. The client states, "I know that my husband will be happy when I'm gone." C. The client insists on eating the whole pizza or not eating any pizza at all. D. The client states, "My father isn't capable of showing me any love or respect."

C. The client insists on eating the whole pizza or not eating any pizza at all.

The clinic nurse has provided instructions to the mother of a child with a urinary tract infection. Which statements by the mother indicates a need for further instruction? Select all that apply. A. "I should increase my child's fluid intake." B. "I should not use bubble baths with my child." C. "I should wipe my child from front to back after urination or a bowel movement." D. "I should encourage my child to hold the urine and to urinate only 4 times each day." E. "I should administer the antibiotics to my child until urinary tract infection symptoms disappear."

D. "I should encourage my child to hold the urine and to urinate only 4 times each day." E. "I should administer the antibiotics to my child until urinary tract infection symptoms disappear."

The nurse is preparing a client for a lumbar puncture. The nurse determines the client needs further teaching if the client makes which statement? A. "I can expect the nurse to look at the site frequently." B. "I will call for the nurse if I experience pain down my legs." C. "I will not be able to sit up for 6 to 8 hours after the procedure." D. "I will be allowed to walk to the bathroom once I return from the procedure."

D. "I will be allowed to walk to the bathroom once I return from the procedure."

Which statement by the client with diabetes mellitus indicates that teaching by the nurse was effective? A. "I will check my hemoglobin A1c each morning before I eat or drink anything." B. "I know my medication is working if I do not have symptoms of hypoglycemia." C. "I must call my health care provider immediately if my blood glucose is over 150 mg/dL (8.3 mmol/L)." D. "I will have a snack about 2 hours after I give myself my Humalog insulin injection."

D. "I will have a snack about 2 hours after I give myself my Humalog insulin injection."

An older woman is admitted to the acute psychiatric unit with a diagnosis of moderate depression. The client is unclean, her hair is uncombed, and she is inappropriately dressed. She is accompanied by her adult daughter who is very upset about her mother's lack of interest in her appearance. The nurse appropriately alleviates the daughter's concern by making which statement? A. "Hygiene is not important to those who are depressed." B. "Client self-esteem needs take priority over appearances." C. "Group peer pressure on the unit will soon have your mother attending to her hygiene needs." D. "The nurses will assist your mother in meeting hygiene needs until she is able to resume self-care."

D. "The nurses will assist your mother in meeting hygiene needs until she is able to resume self-care."

A patient comes to a clinic with a chief complaint of, "My left arm is red and swollen. It hurts badly enough that I couldn't go to work today." The physician orders computer-assisted tomography (CT) scanning of the left upper extremity. The nurse knows the patient understands the reason for the procedure when he states A. "I need to have this done because my arm is broken." B. "The doctor wants me to have this so that the pain will stop." C. "This will tell you what I did to my elbow because I really don't know what happened." D. "This test will help to better determine where the injury actually is and how severe it is."

D. "This test will help to better determine where the injury actually is and how severe it is."

The nurse is assisting in caring for a client with an intracranial pressure (ICP) monitoring device that is placed in the subarachnoid space. The nurse should understand that which reading is abnormal and requires health care provider notification? A. 7 mm Hg B. 12 mm Hg C. 15 mm Hg D. 21 mm Hg

D. 21 mm Hg

Which of the following patients is at higher risk for inflammatory reactions? A. 2-year-old girl with a healthy diet B. 38-year-old man who is obese C. 54-year-old woman in menopause D. 79-year-old man with diabetes

D. 79-year-old man with diabetes

A client with subarachnoid hemorrhage will be taking nimodipine. How should the nurse describe this medication to the client? A. A vasodilator that will dilate cerebral blood vessels B. A β-adrenergic blocker that will decrease blood pressure C. An angiotensin-converting enzyme that will reduce the blood pressure D. A calcium channel blocker that will decrease spasm in cerebral blood vessels

D. A calcium channel blocker that will decrease spasm in cerebral blood vessels

The client sustained a fracture of the right arm in a car crash. Before leaving the emergency department for radiography, the nurse notes a decrease in the client's level of consciousness (LOC) and decides to use the Glasgow Coma Scale (GCS) to further assess the client. What is the rationale for the nurse to use this assessment scale? A. Exposure to radiation may decrease the LOC more. B. The client needs to be conscious to have a radiograph done. C. A more detailed assessment of the client's emotional status is needed. D. A more detailed assessment of the client's neurological status is needed.

D. A more detailed assessment of the client's neurological status is needed.

Which phone call from prenatal clients would require the nurse to immediately notify the health care provider? A. A woman at 36 weeks' pregnancy who notes pedal edema B. A woman at 7 weeks' pregnancy who notes frequency of urination C. A woman at 16 weeks' pregnancy who notes increased white vaginal discharge D. A woman at 32 weeks' pregnancy who notes clear fluid leaking from the vagina

D. A woman at 32 weeks' pregnancy who notes clear fluid leaking from the vagina

The nurse notes that a child with Hirschsprung disease who is scheduled for surgery has inadequate fluid volume. The nurse should plan to implement which intervention to stabilize the child's hydration status before surgery? A. Monitor daily weight. B. Monitor intake and output. C. Administer tap water enemas. D. Administer intravenous fluids and electrolytes.

D. Administer intravenous fluids and electrolytes

Which is a priority component of the management of myasthenia gravis? A. Consuming a low-residue diet B. Wearing an eye patch at night C. Performing muscle-strengthening exercises D. Administering myasthenia medication before meals

D. Administering myasthenia medication before meals

A client arrives at the emergency department with complaints of hives, itching, and difficulty swallowing and says, "My throat feels as though it is closing off." The client states that they were visiting a relative who has two cats and two dogs and believes that he is allergic to cats. The nurse ensures that the client has a patent airway and then prepares the client for which initial intervention? A. Application of ice to the throat B. Administration of normal saline solution C. Administration of an intravenous (IV) glucocorticoid D. Administration of a subcutaneous injection of epinephrine

D. Administration of a subcutaneous injection of epinephrine

A client with a spinal cord injury suddenly complains of a severe, pounding headache. The nurse quickly checks the client and notes that the client is diaphoretic and has an elevated blood pressure and a drop in heart rate. The nurse suspects that the client is experiencing autonomic dysreflexia, elevates the head of the client's bed, and should immediately perform which action? A. Notify the health care provider. B. Increase the rate of intravenous fluids. C. Check to see if the client has a prescription for an antihypertensive. D. Check the client's bladder for distention and the rectum for impaction.

D. Check the client's bladder for distention and the rectum for impaction.

The client with acute pancreatitis has an elevated amylase and lipase that is 5 times the normal value. What assessment finding is most important for the nurse to address? A. Client is lying in a fetal-like position. B. Pain rated "7" in the left upper quadrant C. Gray-blue discoloration around the umbilicus D. Cheek has a muscle spasm when the mastoid is tapped.

D. Cheek has a muscle spasm when the mastoid is tapped.

The nurse is caring for a client with Crohn's disease. Which is the most likely indication of a complication occurring from the disease? A. Erythrocyte sedimentation rate (ESR) is elevated. B. White blood cell (WBC) count is 10,500 cells/mm3 (10.5 × 10/L). C. Client reports abdominal cramping after eating a raw apple. D. Client reports pus in urine and a temperature 100.4° F (38° C).

D. Client reports pus in urine and a temperature 100.4° F (38° C).

The nurse is caring for a client with a diagnosis of cirrhosis of the liver and is monitoring the client for signs of portal hypertension. Which initial sign, if noted in the client, indicates the presence of portal hypertension? A. Weak pulse B. Hypotension C. Flat neck veins D. Crackles on auscultation of the lungs

D. Crackles on auscultation of the lungs

The nurse notes that a client is receiving ganciclovir sodium. The nurse suspects that the client is receiving this medication for the treatment of which disorder? A. Pancreatitis B. Urolithiasis C. Nephrotic syndrome D. Cytomegalovirus retinitis

D. Cytomegalovirus retinitis

The nurse is monitoring a client with a head injury for signs of increased intracranial pressure (ICP). Which finding indicates an early sign of increased ICP? A. Decrease in pulse rate B. Shallow, slow respirations C. Rise in systolic blood pressure D. Decreasing level of consciousness

D. Decreasing level of consciousness

The nurse is caring for a 5-year-old client 3 hours after a tonsillectomy. Which drink should the nurse offer the child to encourage fluid intake? A. Milk B. Orange juice C. Cherry Gatorade D. Diluted apple juice

D. Diluted apple juice

The nurse is giving information at a community session. What is the best advice on how to avoid getting hepatitis C? A. Wash your hands daily. B. Avoid acetaminophen. C. Do not drink water when traveling. D. Do not share intravenous (IV) needles.

D. Do not share intravenous (IV) needles.

The nurse is giving a client instructions about taking ciprofloxacin to treat a urinary tract infection. Which instruction should the nurse provide to the client? A. Drink a glass of milk with the medication. B. Take the medication with an aluminum-based antacid. C. Take the medication with a magnesium-based antacid. D. Drink cranberry juice and other citrus juices during medication therapy.

D. Drink cranberry juice and other citrus juices during medication therapy.

What assessment finding should the nurse expect to note in a client with cirrhosis who has ascites? A. Elevated albumin levels B. Pitting edema in the abdomen C. Serum sodium greater than 150 mEq/L (150 mmol/L) D. Dullness to percussion in the abdomen

D. Dullness to percussion in the abdomen

The nurse is caring for a patient with a diagnosis of multiple sclerosis (MS). The nurse should be aware of which associated response? A. Primary immunodeficiency B. Secondary immunodeficiency C. Optimal immune response D. Exaggerated immune response

D. Exaggerated immune response

The nurse reviews the laboratory results of a client receiving chemotherapy and notes that the white blood cell count is extremely low. The nurse asks a nursing student assigned to care for the client to place the client on neutropenic precautions. The nurse determines the need to review the procedures for neutropenic precautions if the student nurse took which action? A. The water pitcher was removed from the client's room. B. Fresh cut flowers were removed from the client's room. C. A box of face masks was placed at the entrance to the client's room. D. Fresh pears and apples brought to the client by a family member were left in the client's room.

D. Fresh pears and apples brought to the client by a family member were left in the client's room.

The clinic nurse is collecting subjective and objective data from a client and notes that the client is taking abacavir. The nurse determines that this medication has been prescribed to treat which condition? A. Osteoporosis B. Rheumatoid arthritis C. Polycystic kidney disease D. Human immunodeficiency virus (HIV) infection

D. Human immunodeficiency virus (HIV) infection

A client has been newly diagnosed with diabetes mellitus. The nurse should perform which action as the first step in teaching the client about the disorder? A. Decide on the teaching approach. B.Plan for the evaluation of the session. C. Gather all available resource materials. D. Identify the client's knowledge and needs.

D. Identify the client's knowledge and needs.

A client is taking amiloride hydrochloride daily. The nurse should tell the client to take the dose at what time? A. At bedtime B. On an empty stomach C. Between lunch and dinner D. In the morning with breakfast

D. In the morning with breakfast

The client is a 26-year-old who is 12 weeks pregnant and visiting with the nurse in the clinic. Her complete blood cell count shows the hemoglobin to be 10.1 g/dL. What would be the most appropriate nutrition education based on this information? A. Increase the amount of water in the diet. B. Increase the amount of fresh fruits in the diet. C. Increase the amount of milk and yogurt in the diet. D. Increase amounts of meat and dried beans in the diet.

D. Increase amounts of meat and dried beans in the diet.

Intravenous tobramycin sulfate is prescribed for a client with an abdominal wound infection. The nurse should monitor the client for which finding that is indicative of an adverse effect? A. Increased urine output B. Decreased blood pressure C. Decreased serum creatinine D. Increased blood urea nitrogen

D. Increased blood urea nitrogen

The nurse is monitoring the status of a client in labor who is experiencing hypotonic uterine dysfunction. The nurse interprets that which finding would least likely be consistent with this type of dysfunctional labor? A. Contractions weaken during the active stage of labor. B. Contractions become inefficient or stop during the active stage of labor. C. The client initially makes normal progress into the active stage of labor, and then contractions weaken. D. The client is having painful and frequent contractions that are ineffective in causing cervical dilation or effacement to progress.

D. The client is having painful and frequent contractions that are ineffective in causing cervical dilation or effacement to progress.

A client is hospitalized with active tuberculosis (TB) and is being transported to the radiology department. What is the most appropriate action by the nurse to ensure a safe environment? A. The nurse should wear a mask, gown, and gloves. B. No mask is needed because hospital air is circulated frequently. C. Cancel the radiology prescription as the client cannot leave the hospital room. D. The client should wear a high-efficiency particulate air respiratory (HEPA) mask.

D. The client should wear a high-efficiency particulate air respiratory (HEPA) mask.

The nurse is assigned to care for an obstetric client with acquired immunodeficiency syndrome (AIDS). The nurse develops a plan of care for the client and should include which priority client goal in the plan? A. The client moves through the grief process. B. The client knows about local AIDS support groups. C. The client will avoid any sexual relations during the remainder of the pregnancy. D. The client will not develop an opportunistic infection during the remainder of the pregnancy.

D. The client will not develop an opportunistic infection during the remainder of the pregnancy.

The client is in active labor at 38 weeks' pregnancy. She has recently received a continuous lumbar epidural infusion. Which assessment finding would prompt the nurse to request the presence of the anesthesiologist? A. The client is unable to void. B. The client's amniotic sac has broken, and she is leaking clear fluid. C. The client's legs are numb, and she cannot move them. D. The client's blood pressure is 80/40 mm Hg.

D. The client's blood pressure is 80/40 mm Hg.

A client is hospitalized at 28 weeks' pregnancy for preterm labor. The health care provider has prescribed nifedipine 20 mg by mouth every 6 hours. Which assessment finding would require notification of the health care provider? A. The client notes a frontal headache. B. The client reports increased fetal movement. C. The client's blood pressure is 90/50 mm Hg. D. The client's contractions have increased in frequency to every 2 minutes.

D. The client's contractions have increased in frequency to every 2 minutes.

The nurse is caring for a newborn infant at 6 hours of age. The mother had hepatitis B virus infection during pregnancy, and she is positive for hepatitis B surface antigen (HBsAg) and hepatitis B e antigen (HBsAg e). Which practice is recommended to decrease the risks of transmission of hepatitis B to the newborn? A. The mother should not breastfeed. B. The newborn should begin medication to treat hepatitis B. C. The newborn should be placed in isolation until hepatitis B testing is done. D. The newborn should receive hepatitis B vaccine and hepatitis B immunoglobulin.

D. The newborn should receive hepatitis B vaccine and hepatitis B immunoglobulin.

The clinic nurse is checking the neurological status of a client. The nurse would assess for new memory by asking the client about what information? A. To state the city of birth B. What the client ate for lunch yesterday C. What type of transportation was used to get to the clinic D. To repeat three unrelated words spoken to the client immediately and 5 minutes later

D. To repeat three unrelated words spoken to the client immediately and 5 minutes later

The nurse is caring for a hospitalized client with a diagnosis of acute pancreatitis. The nurse should assist the client to which position that will decrease the abdominal pain? A. Prone B. Supine with the legs straight C. Side-lying with the head of the bed flat D. Upright in a sitting position with the trunk flexed

D. Upright in a sitting position with the trunk flexed

When caring for a patient on the neuro-trauma unit, the nurse should assess for which signs and symptoms of increased intracranial pressure? A. Dehydration B. Hunger C. Nausea D. Vomiting

D. Vomiting

Which is an assessment finding in Graves' disease? A. Dry skin B. Excessive sleepiness C. Delayed linear growth D. Weight loss despite excellent appetite

D. Weight loss despite excellent appetite

The nurse is reviewing the laboratory results of a client receiving intravenous (IV) chemotherapy. The nurse should initiate neutropenic precautions if which laboratory result is noted? A. Clotting time of 10 minutes B. Ammonia level of 10 mcg/dL (7 mcmol/L) C. Platelet count of 100,000 cells/mm3 (100 × 109/L) D. White blood cell (WBC) count of 2000 cells/mm3 (2.0 × 109/L)

D. White blood cell (WBC) count of 2000 cells/mm3 (2.0 × 109/L)

A client is diagnosed with diabetic ketoacidosis. On assessment of the client, the nurse expects to note which respiratory pattern that occurs in this condition? A. Regular but abnormally sloq B. labored and increased in depth and rate C. regular but interspersed with periods of apnea D. abnormally deep and regular, with an increased rate

D. abnormally deep and regular, with an increased rate


Kaugnay na mga set ng pag-aaral

PVN 106 Pharm EAQ Practice Test: CV Meds

View Set

NUR 1024: NCLEX Questions for Hip Fractures

View Set

Care for Patients With Musculoskeletal Problems

View Set

Social Entrepreneurship quiz ch. 11-13

View Set

Organizational Behavior Ch 14-16

View Set

Chapter 10: Managing Conflict and Negotiations

View Set